que

69
Que: Can you provide some information on this examination? Ans: The Civil Services Examination (CSE) is conducted every year by the Union Public Service Commission (UPSC) as a common entrance examination for recruitment to various services like the Indian Administrative Service (IAS), Indian Foreign Service (IFS), Indian Police Service (IPS), Group 'A' and Group 'B' - Central Services. (more content follows the advertisement below) A D V E R T I S E M E N T Que: Who can appear for this examination? Ans: I. Academic Eligibility The candidate must possess a degree from an Indian University or an educational institution deemed as a University or possess an equivalent qualification . Those in the final year of a degree course can also appear in the Preliminary Examination. II. Other Eligibility Conditions i. For the IAS and the IPS, the candidate must be a citizen of India. For the other services, a candidate must be either: a citizen of India, or a subject of Nepal, or a subject of Bhutan, or a Tibetan refugee who came over to India before January 1, 1962, with the intention of permanently settling in India, or a person of Indian origin who has migrated from Pakistan, Burma, Sri Lanka, East African countries of Kenya, Uganda, the United Republic of Tanzania, Zambia, Malawi, Zaire, Ethiopia and Vietnam with the intention of permanently settling in India. The candidate must have attained 21 years of age on August 1st of the year of examination and must not have attained 30 years of age on that date. The upper age limit will be relaxed by 3 years for OBC candidates and 5 years for SC/ST candidates. The upper age limit is also relaxed in favour of certain categories of civil servants working under the Government of India and Defence Services Personnel. Que: What are Optional Subjects for the Main Examination?

Upload: hemant-swrnkar

Post on 03-Dec-2014

54 views

Category:

Documents


0 download

TRANSCRIPT

Page 1: Que

Que: Can you provide some information on this examination?Ans: The Civil Services Examination (CSE) is conducted every year by the Union Public Service Commission (UPSC) as a common entrance examination for recruitment to various services like the Indian Administrative Service (IAS), Indian Foreign Service (IFS), Indian Police Service (IPS), Group 'A' and Group 'B' - Central Services.

(more content follows the advertisement below)A D V E R T I S E M E N T

Que: Who can appear for this examination?

Ans: I. Academic Eligibility

The candidate must possess a degree from an Indian University or an educational institution deemed as a University or possess an equivalent qualification. Those in the final year of a degree course can also appear in the Preliminary Examination.

II. Other Eligibility Conditions

i. For the IAS and the IPS, the candidate must be a citizen of India.

For the other services, a candidate must be either:

a citizen of India, or

a subject of Nepal, or

a subject of Bhutan, or

a Tibetan refugee who came over to India before January 1, 1962, with the intention of permanently settling in India, or

a person of Indian origin who has migrated from Pakistan, Burma, Sri Lanka, East African countries of Kenya, Uganda, the United Republic of Tanzania, Zambia, Malawi, Zaire, Ethiopia and Vietnam with the intention of permanently settling in India.

The candidate must have attained 21 years of age on August 1st of the year of examination and must not have attained 30 years of age on that date. The upper age limit will be relaxed by 3 years for OBC candidates and 5 years for SC/ST candidates. The upper age limit is also relaxed in favour of certain categories of civil servants working under the Government of India and Defence Services Personnel. 

Que: What are Optional Subjects for the Main Examination?

 Ans: Optional subjects: Agriculture, Animal Husbandry and Veterinary Science, Botany, Chemistry, Civil Engineering, Commerce and Accountancy, Economics, Electrical Engineering, Geography, Geology, History, Law, Management, Mathematics, Mechanical Engineering, Medical Science, Philosophy, Physics, Political Science and International Relations, Psychology, Public Administration, Sociology, Statistics, Zoology. Each paper is of 3 hours duration. 

Que: Which combinations not allowed  in IAS Examination?

Page 2: Que

 Ans: The following combinations not allowed are: Political Science & International Relations and Public Administration Commerce and Management Anthropology and Sociology Maths and StatisticsAgriculture and Animal Husbandry and Veterinary Science Management and Public Administration Animal Husbandry & Veterinary Science and Medical Science Any two branches of engineering.Combination of two literatures in the above list. 

Que: Information about

all Eligibility Conditions?

Ans: (i) Nationality:

(1) For the Indian Administrative Service and the Indian Police Service, a candidate must be a citizen of India.(2) For other services, a candidate must be either: (a) A citizen of India, or(b) a subject of Nepal, or(c) a subject of Bhutan, or(d) a Tibetan refugee who came over to India before 1st January, 1962 with the intention of permanently settling in India. or (e) a person of Indian origin who has migrated from Pakistan, Burma, Srilanka, East African countries of Kenya, Uganda, the United Republic of Tanzania, Zambia, Malawi, Zaire, Ethiopia and Vietnam with the intention of permanently settling in India.Provided that a candidate belonging to categories (b), (c), (d) and (e) shall be a person in whose favour a certificate of eligibility has been issued by the Government of India.Provided further that candidates belonging to categories (b), (c) and (d) above will not be eligible for appointment to the Indian Foreign Service.A candidate, in whose case a certificate of eligibility is necessary, may be admitted to the examination but the offer of appointment may be given only after the necessary eligibility certificate has been issued to him by the Government of India.

(ii) Minimum Educational Qualifications:The candidate must hold a degree of any of Universities incorporated by an Act of the Central or State Legislature in India or other educational institutions established by an Act of Parliament or declared to be deemed as a University Under Section 3 of the University Grants Commission Act, 1956, or possess an equivalent qualification.

Note I: Candidates who have appeared at an examination the passing of which would render them educationally qualified for the Commission's examination but have not been informed of the results as also the candidates who intend to appear at such a qualifyingexamination will also be eligible for admission to the Preliminary Examination. All candidates who are declared qualified by the Commission for taking the Civil Services (Main) Examination will be required to produce proof of passing the requisite examination with their application for the Main Examination failing which such candidates will not be admitted to the Main Examination.

Note II: In exceptional cases the Union Public Service Commission may treat a candidate who has not any of the foregoing qualifications as a qualified candidate provided that he has passed examination conducted by the other Institutions, the standard of which in the opinion of the Commission justifies his admission to the examination.

Note III: Candidates possessing professional and technical qualifications, which are recognised by Government, as equivalent to professional and technical degree would also be eligible for admission to the examination.

Note IV: Candidates who have passed the final professional M.B.B.S. or any other Medical Examination but have not completed their internship by the time of submission of their applications for the Civil Services (Main) Examination, will be provisionally admitted to theExamination provided they submit along with their application a copy of certificate from the concerned authority of the University/Institution that they had passed the requisite final professional medical examination. In such cases, the candidates will be required to produce at the time of their interview original Degree or a certificate from the concerned competent authority of the University/Institution that they had completed all requirements (including completion of internship) for the award of the Degree.

(iii) Age limit: 21 years must be completed on 1st August of the year, which a candidate is appearing. Maximum 30 for general category, 33 for OBCs and 35 for SCs/STs. Ex-servicemen will get 5 more years exemption from the prescribed

Page 3: Que

age limit.

(The date of birth accepted by the Commission is that entered in the Matriculation or Secondary School Leaving Certificate or in a certificate recognised by an Indian University as equivalent to Matriculation or in an extract from a Register of Matriculates maintained by a University, which extract must be certified by the proper authority of the University or in the Higher Secondary or an equivalentexamination certificate).

(iv) Number of Attempts: Four attempts for open, seven for OBCs and no limit for SCs/STs. If a person appears in the PreliminaryExamination or even one paper is counted as an attempt.

(v) Restrictions on applying for the examination: A candidate who is appointed to the Indian Administrative Service or the Indian Foreign Service on the results of an earlier examinationand continues to be a member of that service will not be eligible to compete at this examination.

Que: How to Apply for the examination?

Ans: The UPSC have developed an application form common for all their examinations, which will be processed on computerised machines. This application form alongwith an Information Brochure containing general instructions for filling up the form, an acknowledgement card and an envelope for sending the application is obtainable from the designated Head Post Offices/Post Offices throughout the country as against cash payment of Rs. 20/- (Rupees twenty only). Form should be purchased from the designated Post Offices only and not from any other agency. This form can be used only once and for only one examination. Candidates must use the form supplied with the Information Brochure only and they should in no case use photocopy / reproduction / unauthorisedly printed copy of the Form. Since this form is electronically scannable, due care should be taken to fill up the application form, correctly. While filling up the application form, please refer to detailed instructions given in the Notice.

Keywords:

Frequently Asked Questions for IAS Aspirants,Academic Eligibility,science questions, questions books, questions notes, paper questions, examination questions, exam questions, sample questions, tests questions, study questions

A D V E R T I S E M E N T

1. Be very particular about the subject you choose for prelims, as you will be appearing for an objective type of paper. History, maths, geography may prove to be very scoring. Choose subjects which have availability of books, reading material and guidance. In recent years engineering subjects like civil and electrical can be chosen, giving BEs and IITians an edge (yes! even here they are giving the BAs and BScs a tough fight!)

(more content follows the advertisement below)A D V E R T I S E M E N T

2. G.K. will definitely pay in your prelims. Reading newspapers, watching TV news and of course quiz shows like KBC is a must.

3. If you have been lucky enough to reach the interview stage book knowledge may not be the only thing you need. Your mental alertness will count as they ask you questions like “How many steps did you walk up to reach here?” or “ What is the colour of the wall behind you ?” - So be prepared.

4. Enhance your personality because it will definitely be one of the criteria for selection. For IPS physical wellbeing is of great importance, you should be medically fit.

5. Improve your communication skills. IFS aspirants must be proficient in at least one foreign language. So go ahead, have your say in this political mess of our country and try to make it a better place.

Page 4: Que

TIPS on IAS From Mr. Vaid of Vaid's ICS Coaching CenterIt was in 1981 when I had my first encounters with UPSC through the aspirants (for IAS etc.) who sought my help to prepare for certain specific topics in GS and my parent discipline, i.e. Anthropology. Today, 13 years later, through regular interaction with the UPSC experts and students all over India, particularly at Delhi, Lucknow, Patna, Chandigarh and Hyderabad, I find myself in a favourable position to pen down my experiences and suggestions to the young aspirants. In the early eighties, there were three (Delhi, Allahabad and Patna) major centre of UPSC related activities, while Hyderabad was coming up fast. There used to be 40 to 60 thousand contenders for the coveted posts. Today, while the number of aspirants has risen by over 200 percent, many new centres have come up in almost every state capital and many other major cities. In those days professionals did not see any merit in contention for these jobs, but recent years have witnessed a flood of such aspirants, at times sweeping the top positions in the merit list. In those days History, Sociology, Political Science etc. used to be the most popular optionals while today, many hitherto less known subjects have taken over. Besides, many states have changed their pattern of PCS examination in line with UPSC examination. This change in the scenario, resultant of a number of factors, demands that the serious contenders to the job must adopt a more rational, more systematic strategy and approach, right from the moment they think of preparing for the civil services examination.

On the first thought of appearing in the Civil Services Examination (C.S.E.), the first issue is the selection of optionals. In a way, selection of right optionals is the most crucial step. Post-graduates in any discipline, generally prefer taking up their parent subject for Prelims. Though such selection is quite logical, aspirants with any science discipline as their base subject must ponder over the issue more seriously. Maths, Physics, Geology etc., optionals for Prelims must be opted for, only if one is out and out thorough with the subject. As UPSC ensures that 'some' candidates from every discipline must qualify for the Mains, aspirants with a science subject must judge themselves correctly regarding their own capability to be included in these 'some'. Some of the social sciences (e.g. History, Sociology and Public Administration) opted by quite a significant majority can, however, be a much safer bet, particularly for those (ordinary science graduates) who do not have any specific parent subject or for those who are not able to develop confidence in their parental subject. In either case, the formal preparation must start from the basic fundamentals, even if a person is already a post-graduate in that subject.

General Studies (GS) has to be prepared on quite a different plane because the content of this paper is too spread out and because any issue, major or minor, can be asked objectively. Traditionally, 10 + 2 level books of NCERT or any state education board had been covering a major part of the content but, these days, the items in news during the last one year or so, are being asked quite frequently. Problems are generally faced in everyday science (by the candidates with social sciences background) or in mental ability or in the fundamentals of economics. The most important aspect for the preparation of GS Prelims, therefore, is to identify the loop-holes and plug them urgently.

Many of you prepare a subject in terms of "reading" (once, twice, thrice etc.) with or without underlining the important facts. It results not only in longer time of preparation, but also many important points may skip. Also, even if you underline the highlighting points, most of you have the tendency to read the text in full while going for the subsequent readings. It is, therefore, advisable that texts must be read once and all important points (likely to be forgotten) recorded separately so that you need not study volumious texts again. This would save on your time and should result in greater efficiency. The Mains: For right approach, the preparation of Mains should start before or at least concurrent with Prelims. Just because you have to prepare for Essay, GS and the two optionals (English and a regional language, the one/two other compulsory, do not require a separate preparation) you never get enough time (to prepare for all these) after the declaration of Prelims' results. Further, while Prelims is only a screening test, it is on the basis of your performance in Mains, on which mainly depends the final outcome of your efforts. Strategically, therefore, the preparation for the examination should start about one year in advance and you should think about appearing in the Prelims only when you have had a strong grip over the Mains' subject matter.

Keywords: 

Tips for IAS Exams, tips for ias exam, multiple choice exam, computer science exam, exam study guide, objective exam, exam notes, physics exam, paper exam, exam books, biology exam, math exam, chemistry exam, exam papers, sample exam, exam preparation, exam answers, exam quiz, test exam, study exam, exam question

Civil Service Test PracticeBy Ajit kumar singh on July 21st, 2011

Page 5: Que

No matter how much you may want one of these pre-employment types of exams to go away, it never will. And the best way to deal with an examination is to embrace it and give it your best shot. After all, careful preparation is required. And this type of pre-test prep can help you [...]

No Comments | Read full story »

Current Affairs Careers in IAS IAS Eligibility IAS Interview IAS Centers List

IAS Papers : Tips to Prepare General Studies PaperAn extensive study is required for preparing GS Prelims. Start your preparation with ncert books of 10th standard on various subjects like History, Geography, Polity, Science etc. and then go through other books recommended for G.S. It will be better to prepare G.S. Mains first and then for Prelims.For G.S. Mains start your preparation [...]

Current AffairsThe current affairs of india and international current event affairs, are an important part of the general studies paper, both in the mains and the prelims. At the interview current affairs news rules and decides the rank of the candidate and even whether s/he will get through or not. In such a scenario the [...]

Multiple Choice Economics Questions1.     According to the Economic survey 2010-11 what percentage growth was recorded by the manufacturing sector in 2010?a.    9.1%b.    8.6%c.    7.3%d.    8%Answer: a2.     As per the economic Survey of India 2010-11, the production of food grains is estimated at over 232 million tonnes with record production of which food crop?a.    Riceb.    Wheatc.    Bajrad.    MaizeAnswer: [...]General Knowledge Questions With Answers1.Which Indian state is the largest producer in the world of the golden coloured ‘Muga’ silk?AssamOrissaWest Bengal2. Anandmath’, ‘Durgeshnondini’ and ‘Kapalkundala’ novels were authored by which Bengali writer ?Sharat Chandra ChatterjeeBankim Chandra ChatterjeeBibhuti Bhushan Bandopadhyay3. Which country is also known as the ‘Rainbow Nation’ ?AustraliaSwitzerlandSouth Africa4. Which among the following is manufactured at Avadi [...]IAS Exam Preparation Guidance and Success Tips

Page 6: Que

Exam fear is an awful sensation that we many people experience. From those early school examinations we sat, right through to the Civil Service test – examinations never get any easier. But there is one thing you can do though to give yourself the best chance possible at getting a great mark. And that [...]UPSC aspirants can now choose Indian languages for interviewThe Union Public Service Commission (UPSC) has informed the Bombay high court that candidates, who opt for an Indian language medium (other than Hindi) for the written Civil Services exam, can henceforth either choose the same Indian language, English or Hindi for the interview.Similarly, even those candidates, who give the main exam in English, [...]3 Tihar inmates appear in civil services examAmong the thousands of civil services aspirants are three inmates of Tihar Prisons, including a murder convict and an IITian, who appeared in the examination this time.Siddharth Singh Sidar from Chhattisgarh, who is undergoing imprisonment for cheating and once trained IAS aspirants, Amit Jha from Bihar who was held for kidnapping and Sandeep from [...]How to crack civil service examCivil services exams are one of the respectable exams. Each year, thousands of people take up this exam but only few are able to make it through preliminary exams. Cracking IAS examination is not a cakewalk. Aspiring candidates have to undergo a rigorous process for clearing the paper and being selected for preferred services. [...]

Resources BCA Papers Best Online Articles CBSE Papers Chartered Accountancy Course Commonwealth Games 2010 Entrance Exams in India Exam Papers Free Article Directory GPS Vehicle Tracking System IGNOU Papers Indian TOp Schools Live India MBA Papers MCA Papers Quiz Bank RSCS

Latest Update Civil Service Test Practice Multiple Choice Economics Questions General Knowledge Questions With Answers IAS Exam Preparation Guidance and Success Tips UPSC aspirants can now choose Indian languages for interview 3 Tihar inmates appear in civil services exam How to crack civil service exam CSAT Exam 2011 Candidates prove aptitude for Civil Services Solved General Knowledge Questions

Latest Comments Radhika on IAS Papers : English Syllabus for IAS Exam gokul singh bisht on Welcome To www.iaspapers.info

Page 7: Que

HIMANSHU on IAS Papers : General Studies 2003 (Main) sanat on IAS Papers : How to choose Optional Subject mohit kumar on IAS Papers : General Studies Preliminary Exam 2009

 

"Do you know GK can help you in cracking the IAS Exams. For Better Performance in Indian Administrative Service (IAS) Exam use our Quiz Master for practice."

    Go for Practice   

Latest Popular Topics Archives Civil Service Test Practice Multiple Choice Economics Questions General Knowledge Questions With Answers IAS Exam Preparation Guidance and Success Tips UPSC aspirants can now choose Indian languages for interview 3 Tihar inmates appear in civil services exam How to crack civil service exam CSAT Exam 2011 Candidates prove aptitude for Civil Services Solved General Knowledge Questions IAS Papers : Solved Multiple Choice Questions for Indian History IAS Preparation in a few simple methods. Solved Sociology Questions for IAS Papers General Knowledge Questions for IAS Preparation General Ability Questions HISTORY (MAIN) : 2004

Go

Page 8: Que

General Knowledge about Indian freedom struggle IAS Papers : COMMERCE AND ACCOUNTANCY PAPER IAS Papers : General Knowledge Questions Quiz What Is the IAS Exam Pattern? Books for IAS CSAT Exam CSAT For UPSC Civil Services Exam UPSC Exam Preparation Previous Exam Papers Will Assist You In Knowing The Exam Pattern Exam Pattern of Civil Service Aptitude Test The Reality of The Indian Civil Services Examination Steps to Crack IAS Paper CSAT Is The New Civil Services UPSC Civil Services (Pre) Examination-2011 How To Prepare For IAS Exam

TagsIAS Papers IAS Civil Services IAS ExamIAS Preparation IAS Syllabus Civil Service Exam IAS Exam Pattern IAS Exam Syllabus General Studies IAS Forum IAS Eligibility Criteria Time Management UPSCHow to Prepare for IAS IAS Overview Career in IAS IAS Current Affairs IAS Tips IAS Application Form CentersIAS Regulations Cut off Marks in IAS Economics Method of Making Notes IAS

Preparation Strategy CSAT IAS Exams Style of Answering Questions Postings of an IAS Officer Civil Services

Exam IAS Main Exam civil servicePublic Administration Civil Services Aptitude TestInterview Tips for IAS Nature of

Work Training Institutes for IAS Jobs Offered in IAS Exam PreparationSections Under IAS Examinations Sample IAS

Exam PaperInterview Techniques for IAS Calendar for IAS ExamPost Offices for IAS Exam Form Time Table for IAS Exam

 Copyright © 2011 IAS Papers | Civil Service Exam Preparation |

TUESDAY, JUNE 21, 2011

Some important twisted questions asked in IAS Interview.........

And now these are IAS i.e. UPSC Exam 1998 Interview Question and there Answer given by candidates..........oh sorry!! IAS Officers now

.01 Q. How can you drop a raw egg onto a concrete floor without cracking it?A. Concrete floors are very hard to crack! (UPSC Topper)

.02 Q. If it took eight men ten hours to build a wall, how long would it take four men to build it?A. No time at all it is already built. (UPSC 23 Rank Opted for IFS)

.03 Q. If you had three apples and four oranges in one hand and four apples and three oranges in the other hand, what would you have?A. Very large hands.(Good one) (UPSC 11 Rank Opted for IPS)

.04 Q. How can you lift an elephant with one hand?A. It is not a problem, since you will never find an elephant with onehand.(UPSC Rank 14 Opted for IES)

.05 Q. How can a man go eight days without sleep?A. No Probs, He sleeps at night. (UPSC IAS Rank 98)

Page 9: Que

.06 Q. If you throw a red stone into the blue sea what it will become?A. It will Wet or Sink as simple as that. (UPSC IAS Rank 2)

.07 Q. What looks like half apple?A: The other half. (UPSC - IAS Topper)

.08 Q. What can you never eat for breakfast?A: Dinner.

.09 Q. What happened when wheel was invented?A: It caused a revolution.

.10 Q. Bay of Bengal is in which state?A: Liquid (UPSC 33Rank)

.11 Q: what is the opposite of Nag panchmi?A: Nag did not punch me.

50 The national flower of Italy is Lily

51 The national flower of China is Narcissus

52 The permanent secretariat of the SAARC is located at Kathmandu

53 The gateway to the Gulf of Iran is Strait of Hormuz

54 The first Industrial Revolution took place in England

55 World Environment Day is observed on 5th June

56 The first Republican President of America was Abraham Lincoln

57 The country famous for Samba dance is Brazil

58 The name of Alexander's horse was Beucephalus

59 Singapore was founded by Sir Thomas Stamford Raffles

60 The famous British one-eyed Admiral was Nelson

61 The earlier name of Sri Lanka was Ceylon

62 The UNO was formed in the year 1945

63 UNO stands for United Nations Organization

64 The independence day of South Korea is celebrated on 15th August

65 `Last Judgement' was the first painting of an Italian painter named Michelangelo

66 Paradise Regained was written by John Milton

67 The first President of Egypt was Mohammed Nequib

68 The first man to reach North Pole was Rear Peary

69 The most famous painting of Pablo Picasso was Guermica

70 The primary producer of newsprint in the world is Canada

71 The first explorer to reach the South Pole was Cap. Ronald Amundson

72 The person who is called the father of modern Italy is G.Garibaldi

73 World literacy day is celebrated on 8th September

Page 10: Que

74 The founder of modern Germany is Bismarck

75 The country known as the land of the midnight sun is Norway

76 The place known as the Roof of the world is Tibet

77 The founder of the Chinese Republic was San Yat Sen

78 The first Pakistani to receive the Nobel Prize was Abdul Salam

79 The first woman Prime Minister of Britain was Margaret Thatcher

80 The first Secretary General of the UNO was Trygve Lie

81 The sculptor of the statue of Liberty was Frederick Auguste Bartholdi

82 The port of Banku is situated in Azerbaijan

83 John F Kennedy was assassinated by Lee Harry Oswald

84 The largest river in France is Lore

85 The Queen of England who married her brother-in-law was Catherine of Aragon

86 The first negro to be awarded the Nobel Peace Prize was Ralph Johnson Bunche

87 The first British University to admit women for degree courses was London University

88 The principal export of Jamaica is Sugar

89 New York is popularly known as the city of Skyscrapers

90 Madagascar is popularly known as the Island of Cloves

91 The country known as the Land of White Elephant is Thailand

92 The country known as the Land of Morning Calm is Korea

93 The country known as the Land of Thunderbolts is Bhutan

94 The highest waterfalls in the world is the Salto Angel Falls, Venezuela

95 The largest library in the world is the United States Library of Congress, Washington DC

PREVIOUS PAGE

1. Preliminary examinationsThe prelims are held around May-June.It has objective ie multiple choice questions. There are two papers (duration of each being 2 hrs).

a) General studiesb) Optional subject

(more content follows the advertisement below)A D V E R T I S E M E N T

The prelims are held around May-June.It has objective ie multiple choice questions. There are two papers (duration of each being 2 hrs). a) General studies and b)Optional subject: you can choose from Psychology, Sociology, Philosophy, Agriculture, Maths, Phy, Chem, Stats etc. Important: It is not compulsory to choose

Page 11: Que

the same subjects of your graduation. The results are declared in July-August and the successful candidates are called for the main examinations. PS. The prelims score has no bearing for determining the final merit list for the services.

2. Main examinationThe main written examination usually takes place in Oct-Nov, it comprises of 9 papers in all (a lot more strenuous than CAT!) Duration of each paper is 3 hrs. Language: one of the Indian languages, English, Essay GK and Current Affairs: 2 papers on general studiesOptionals: papers from two subjects that fall in optional-1 and two papers from optional-2 (a choice of subjects is listed by the UPSC in the prospectus) Candidates who are successful in the main examinations are eligible for an interview which plays a crucial part in the selection.

The main written examination usually takes place in Oct-Nov, it comprises of 9 papers in all (a lot more strenuous than CAT!) Duration of each paper is 3 hrs. Language: one of the Indian languages, English, Essay GK and Current Affairs: 2 papers on general studiesOptionals: papers from two subjects that fall in optional-1 and two papers from optional-2 (a choice of subjects is listed by the UPSC in the prospectus) Candidates who are successful in the main examinations are eligible for an interview which plays a crucial part in the selection.

3. The InterviewThe interview calls are sent in March-April of the following year and they take place in the month of April-May. The Board of interviewers look for a well-adjusted personality with the social traits, integrity and qualities of leadership . PS. The number of candidates called for the interview is normally twice the number of vacancies.

The interview calls are sent in March-April of the following year and they take place in the month of April-May. The Board of interviewers look for a well-adjusted personality with the social traits, integrity and qualities of leadership . PS. The number of candidates called for the interview is normally twice the number of vacancies.

Here is a list of subjects for IAS/UPSC exams. Under each subject is provided reources regarding the subject that includes prelims and mains IAS syllabus as well as question papers, strategy and suggested readings for the subject. The IAS Syllabus and other services like the IPS and Income Tax is the same.

 

Part A - UPSC Preliminary Examination - Optional Subjects General Studies

Agriculture

Animal Husbandry and Veterinary Science

Botany

Chemistry

Civil Engineering

Commerce

Economics

Electrical Engineering

Geography

Geology

Mathematics

Mechanical Engineering

Medical Science

Philosophy

Physics

Political Science

Psychology

Public Administration

Sociology

Statistics

Zoology

Page 12: Que

Indian History

Law

Part B - UPSC Main Examination - Optional Subjects General Studies

Agriculture

Animal Husbandry and Veterinary Science

Botany

Chemistry

Civil Engineering

Commerce

Economics

Electrical Engineering

Geography

Geology

Indian History

Law

Mathematics

Mechanical Engineering

Medical Science

Philosophy

Physics

Political Science

Psychology

Public Administration

Sociology

Statistics

Zoology

Anthropology

Arabic

Assamese

Bengali

Chinese

English

German

Gujarati

Hindi

Kannada

Kashmiri

Konkani

Malayalam

Manipuri

Marathi

Nepali

Oriya

Pali

Persian

Punjabi

Russian

Sanskrit

Sindhi

Tamil

Telugu

Urdu

Management

Page 13: Que

Keywords:

IAS Exam Syllabus,objective exam, syllabus questions, physics syllabus, chemistry syllabus, mathematics syllabus, physics exam, paper exam, exam books, engineering syllabus, chemistry exam, exam preparation, exam papers, sample exam, science syllabus, study exam, exam questions, science exam

The Main examination comprises of nine papers, all of them of a descriptive type conducted within a span of 20 days.

It is designed to test not only the academic talent of the aspirant but also his ability to coordinate and present his knowledge in a clear and cogent manner. About 12,000 peoples were qualified for the Main Examination in 2010.

Civil Services Main Examination Timetable

Part B - Main Examination - Optional Subjects

Paper-I

One of the Indian Languages to be selected by the candidate from the 22 languages included in the VIIIth Schedule to the Constitution (Qualifying Paper)

300 Marks

Paper-II English (Qualifying Paper) 300 Marks

Paper-III Essay [in the medium you choose] 200 Marks

Papers IV & V

General Studies (300 Marks for each paper)

600 Marks

Papers VI, VII, VIII & IX

Any two subjects (each having 2 papers) to be selected from the prescribed optional subjects (300 marks for each paper)

1200 Marks

Total Marks for Written Examination 2000 Marks

Interview Test [in the medium you choose] 300 Marks

Grand Total 2300 Marks

General Studies

Essay

English Qualifying

Bengali Main Syllabus

Chinese Main Syllabus

English Main Syllabus

Page 14: Que

Indian Languages

Agriculture Main Syllabus

Animal Husbandry & Veterinary Science

Botany Main Syllabus

Chemistry Main Syllabus

Civil Engineering Main Syllabus

Commerce Main Syllabus

Economics Main Syllabus

Electrical Engineering Main Syllabus

Geography Main Syllabus

Geology Main Syllabus

Indian History Main Syllabus

Law Main Syllabus

Mathematics Main Syllabus

Mechanical Engineering Main Syllabus

Medical Science Main Syllabus

Philosophy Main Syllabus

Physics Main Syllabus

Political Science Main Syllabus

Psychology Main Syllabus

Public Administration Main Syllabus

Sociology Main Syllabus

Statistics Main Syllabus

Zoology Main Syllabus

Anthropology Main Syllabus

French Main Syllabus

German Main Syllabus

Gujarati Main Syllabus

Hindi Main Syllabus

Kannada Main Syllabus

Kashmiri Main Syllabus

Konkani Main Syllabus

Malayalam Main Syllabus

Manipuri Main Syllabus

Marathi Main Syllabus

Nepali Main Syllabus

Oriya Main Syllabus

Pali Main Syllabus

Persian Main Syllabus

Punjabi Main Syllabus

Russian Main Syllabus

Sanskrit Main Syllabus

Sindhi Main Syllabus

Tamil Main Syllabus

Telugu Main Syllabus

Urdu Main Syllabus

Management Main

Page 15: Que

Arabic Main Syllabus

Assamese Main Syllabus

IAS CSAT 2011 Solved Paper (General Studies- II)

IAS CSAT 2011 Solved Paper (General Studies- II)Sub Date: 6/12/2011

IAS Prelims General Studies II Solved Question Paper 2011. UPSC conducted Civil Services Preliminary Examination 2011 on 12 June 2011. Go through these solved questions to prepare yourself for major competitive exams conducted by UPSC, SSC, RBI, SBI and other major bodies of the Government of India.

Directions for the following 8 (eight) items:

Read each of the following two passages and answer the items that follow. Your answers to these items should be based on the passages only.

Passage-I

For achieving inclusive growth there is-a critical need to rethink the-role of the State. The early debate among economists about the size of the Goverment can be misleading. The need of the hour is to have an enabling Government. India is too large and complex a nation for the State to be able to deliver all that is needed. Asking the Government to produse all the essential goods, create all the necessary jobs, and keep a curb on the prices of all goods is to lead to a large cumbersome bureaucracy and widespread corruption. The aim must be to stay with the objective of inclusive growth that was laid down by the founding fathers of the nation and also to take a more modern view of what the State can realistically deliver. This is what leads to the idea of an enabling State, that is, a Government that does not try to directly deliver to the citizens everything that they need. Instead, it (1) creates an enabling ethos for the market so that individual enterprise can flourish and citizens can, for the most part, provide for the needs of one another, and (2) steps in to help those who do not manage to do well for themselves, for there will always be individuals, no matter what the system, who need support and help. Hence we need a Government that, when it comes to the market, sets effective, incentive-compatible rules and remains on the sidelines with minimal interference, and, at the same time, plays an important role in directly helping the poor by ensuring  that they get basic education and health services and receive adequate  nutrition and food .

1. According to passage :

1. The objective of inclusive growth was laid down by the founding fathers of the nation.2. Need of the hour is to have an enabling Government.3. The Government should engage III maximum interference in market processes.4. There is a need to change the size of the Government.

Which of the statements given above are correct ?

(a) 1 and 2 only(b) 2 and 3 only(c) 1 and 4 only(d) 1, 2, 3 and 4

Ans: a

2. According to the passage, the strategy of inclusive growth can be effected by focussing on

Page 16: Que

(a) meeting all the needs of every citizen in the country.(b) Increasing the regulations over the manufacturing sector.(c) Controlling the distribution of manufacturing goods.(d) Delivery of the basic services to the deprived sections of the society.

Ans: d

3. What constitutes an enabling Government?

1. A large bureaucracy.2. Implementationof welfare programmes through representatives.3. Creating an ethos that helps individual enterprise4. Providing resources to those who are underprivileged.5. Offering direct help to the poor regarding basic services.

Select the correct answer from the codes given below :

(a) 1, 2 and 3 only(b) 4 and 5 only(c) 3, 4 and 5 only(d) 1, 2, 3, 4 and 5

Ans: c

4. Why is the State unable to deliver "all thai is needed"?

1. It does not. have sufficient bureaucracy.2. It does not promote inclusive growth.

Select the correct answer from the codes given below :

(a) 1 only(b) 2 only(c) Both 1 and 2(d) Neither 1 nor 2

Ans: d

5. What is essential message being conveyed by the author of the passage ?

(a) The objectives of inclusive growth laid down by the foundings fathers of the nation should be remembered.(b) The Government needs to make available more schools and health sevices.(c) The Government needs to establish markets and industries to meet the needs of the poor strata of the society.(d) There is a need to rethink the role of the State in achieving inclusive growth.

Ans: d

Passage-2

The concept of 'creative society' refers to a phase of development of a society in which a large number of potential contradictions become articulate and active. This is most evident when oppressed social groups get politically mobilised and demand their rights. The upsurge of the peasants and tribals, the movements for regional autonomy and self-determination, the environmental movements, and the women's movements in the developing countries are signs of emergence of creative society in contemporary times. The forms of social movements and their intensity may vary from country to country and place to place within a country. But the very presence of movements for social transformation in various spheres of a society indicates the emergence of a creative society in a country.

6. What does the author imply by "creative society" ?

1. A society where diverse art forms and literary writings seck incentive.

Page 17: Que

2. A society where social inequalities are accepted as the norm.3. A society where a large number of contradictions are recognised.4. A society where' the exploited and the oppressed groups grow conscious of. their human rights and upliftment.

Select the correct answerusing the codes given below :

(a) 1, 2 and 3(b) 4 only(c) 3 and 4(d) 2 and 4

Ans: c

7. What according to the passage are the manifestations of social movements?

1. Aggressiveness and being incendiary.2. Instigation by external forces.3. Quest for social equality and individual freedom.4. Urge for granting privileges and self-respect to disparaged sections of the society.

Select the correct answer using the codes given below:(a) 1 and 3 only(b) 2 and 4 only(c) 3 and 4 only(d) 1, 2, 3 and 4Ans: c8. With reference to the passage. consider the following statements:1. To be a creative society, it is essential to have a variety of social movements.2. To be a creative society, it is imperative to have potential contradictions and conflicts.Which of the statements given above is/are correct ?(a) 1 only(b) 2 only(c) Both 1 and 2(d) Neither 1 nor 2Ans: c9. Consider the following three statements:1. Only students can participate in the race.2. Some participants in the race are girls.                3. All girl participants in the race are invited for coaching.Which one of the following conclusions can be drawn from the above statements?(a) All participants in the race are invited for coaching.(b) All students are invited for coaching.                (c) All participants in the race are students.(d) None of the statements (a), (b) and (c) given above is correct.

Ans: cDirections for the following 2 (two) items:

Each of the following two items consists of four statements. Of these four statements, two cannot both be true, but both can be false. Study the statements carefully and identify the two that satisfy the above condition. Select the correct answer using the codes given below each set of statements :10. Examine the following statements:

1. All animals are carnivorous.2. Some animals are not carnivorous.3. Animals are not carnivorous.4. Some animals are carnivorous.Codes:(a) 1 and 3(b) 1 and 2(c) 2 and 3(d) 3 and 4Ans: a11. Examine the following statements:

Page 18: Que

1. All trains are run by diesel engine.2. Some trains are run by diesel engine.3. No train is run by diesel engine.4. Some trains are not run by diesel engine.Codes:(a) 1 and 2(b) 2 and 3(c) 1 and 3(d) 1 and 4Ans: c12. Consider the four age pyramids given below namely A, B, C and D representing four different countries.

Which one of them indicates the declining population ?(a) A(b) B(c) C(d) DAns: c13. The followings figures has four curves namely A, B, C and D, Study the figure and answer the item that follows.

Which curve indicates the exponential growth ?(a) A(b) B(c) C(d) DAns: cDirections for the following 2 (two) items:The following pie charts show the break-up of disease categories recorded in the patients from two towns, Town A and Town B. Pie charts plot the disease Categories as percentage of the total number of patients. Based on these, answer the two items that follow the charts.

Page 19: Que

14. Which of the two towns has a higher number of persons with Diabetes?(a) Town A(b) Town B(c) Same in Town A and Town B(d) No inference can be drawnAns: d15. What can we say about persons with more than one disease from these graphs ?(a) There are likely to be persons with more than one disease in Town A.(b) There are likely to be persons with more than one disease in Town B.(c) There'are likely to be persons with more than one disease in both Towns A and B.(d) No inference can be drawn.Ans: b16. Consider the followmg Velocity-Time graph. It shows two trains starting simultaneously on parallel tracks.

With reference to the above graph, which one of the following statements is not correct ?

(a) Train B has an initial acceleration greater than that of Train A.(b) Train B is faster than Train A at all times.(c)  Both trains have the same velocity at time to'(d) Both trains travel the same distance in time to units.Ans: d

Directions for the following 6 (six) items:

Read each of the following two passages and answer the items that follow. Your answers to these items should be based on the passages only.Passage-1

Ecosystems provide people with a variety of goods and services; food, clean water, clean air, flood control, soil stabilization, pollination, climate regulation, spritual fulfilment and aesthetic enjoyment, to name just a few. Most of these benefits either are irreplaceable or the technology necessary to replace them is prohibitively expensive. For example, potable fresh water can be provided by desalinating sea-water, but pnly great cost. The rapidly expanding human population has greatly modified the Earth's ecosystems to meet their increased requirements of some of the goods and services, particularly food, fresh water, timber,  fibre and fuel. These modifications have contributed substantially to human well being and economic development. The benefits have not been equally distributed. Some people have

Page 20: Que

actually been harmed by these changes. Moreover, short-term increases in some ecosystem goods and services have come at the cost of the long-term degradation of others. For example, efforts to increase the production of food and fibre have decreased the ability of some ecosystems to provide clean water, regulate flooding and support biodiversity.

17. With reference to the passage, consider the following statements.Expanding human' population has an adverse effect on :

1. Spiritual fulfilment2. Aesthetic enjoyment3. Potable fresh water4. Production of food and fibre5. Biodiversity

Which of the statements given above are correct ?

(a) 1, 2 and 3 only(b) 2, 4 and 5 only(c) 3 and 5 only(d) 1, 2, 3, 4 and 5

Ans: c

18. The passage mentions that "some people have actually been harmed by these changes." What does it imply ?

1. The rapid expansion of population has adversely affected some people.2. Sufficient efforts, have not been made to increase the production of food and fibre.3. In the short term some people may be harmed, but in the long term everyone will benefit from modifications In the Earth's ecosystems.Which of the statements given above is/are correct?(a) 1 only(b) 2(c) 1 and 3(d) None of the statements given aboveAns: a19. With reference to the passage, consider the following statements:

1. It is imperative to modify the Earth's ecosystems for the well being of mankind.2. Technology can never replace all the goods and services provided by ecosystems.Which of the statements given above is/are correct ?(a) 1 only(b) 2 only(c) Both 1 and 2(d) Neither 1 nor 2Ans: b

Passage-B:A moral act must be our own act; must spring from our own will. If we act mechanically, these is no moral content in our act. Such action would be moral, If we think it proper to act like a machine and do so. For, in doing so, we use our discrimination. We should bear in mind the distinction between acting mechanically and acting intentionally. It may be moral of a king to pardon a culprit. But the messenger carrying out the order of pardon plays only a mechanical part in the king's moral act. But if the messenger were to carry out the king's order considering it to be his duty, his action would be a moral one. How can a man understand morality who does not use his own intelligence and power of thought, but lets himself be swept along like a log of wood by a current ? Sometimes a. man defies convention and acts on his own with a view to absolute good.20. Which of the following statements best describe/describes the thought of the writer?1. A moral act calls for using our discretion.2. Man should react to a situation immediately.3. Man must do his duty.4. Man should be able to defy convention in order to be moral.Select the correct answer from the codes given below :(a) 1 only

Page 21: Que

(b) 1 and 3(c) 2 and 3(d) 1 and 4Ans: d21. Which of the following statements is the nearest definition of moral action, according to the writer ?(a) it is a mechanical action based on official orders from superiors.(b) It is an action baased on our sence of discretion.(c) IS a clever action based on the clarity of purpose.(d) It is a religious action based on understanding.Ans: b22. The passage contains a statement "lets himself be swept along like a log of wood by a current." Among the following statements, which is/are nearest in meaning to this?1. A person docs not use his own reason.2. He is susceptible to influence/pressure.3. He cannot withstand difficulties/challenges.4. He is like a log of wood.Select the correct answer using the codes given below:(a) 1 only(b) 1 and 2(c) 2 and 3(d) 1 and 4Ans: b23. Consider the following distance - time graph. The graph shows three athletes A, Band C running side by side for a 30 km race.

With reference to the above graph consider the following statements :1. the race was won by A.2. B was ahead of A up to 25 km 26 mark.3. C ran very slowly from the begining.Which of the statements given above is/are correct ?(a) 1 only(b) 1 and 2 only(c) 2 and 3 only(d) 1, 2 and 3Ans: b24. Consider the following figures :

What is the missing number ?(a) 7(b) 8(c) 9(d) 10Ans: c25. Study the following figure:

Page 22: Que

A person goes from A to B always moving to the right or downwards along the Jines. How many different routes can he adopt?Select the correct answer from the codes given below :(a) 4(b) 5(c) 6(d) 7Ans: c26. Consider the following figure and answer the item that follows :

What is total number of triangles in the above grid ?(a) 27(b) 26(c) 23(d) 22Ans: cDirections for the following 4 (four) items:

Read the following passage and answer the items that follow. Your answers to these items should be based on the passage only.PassageA country under foreign domination seeks escape from the present in dreams of a vanished age, and finds consolation in visions of past greatness. That is a foolish and dangerous pastime in which many of us indulge. An equally questionable practice for us in India is to imagine that we are still spiritually great though we have come down, in the world in other respects. Spiritual or any other greatness cannot be founded on lack of freedom and opportunity, or on starvation and misery. Many western  writers have encouraged that notion that Indians are other-worldly. I suppose the poor and unfortunate in every country become to some extent other-worldly, unless they become revolutionaries, for this world is evidently not meant for them. So also subject peoples.As a man grows to maturity he is not entirely engrossed in, or satisfied with, the external objective world. He seeks also some inner meaning, some psychological and physical satisfactions. So also with peoples and civilizations as they mature and grow adult. Every civilization and every people exhibit these parallel streams of an external life and an internal life. Where they meet or keep close to each other, there is an equilibrium and stability. When they diverge conflict arises and the crises that torture the mind and spirit.27. The passage mentions that "this world is evidently not meant for them". It refers to people who1. seek freedom from foreign domination.2. live in starvation and misery.3. become revolutionaries .Which of the statements given above is/are correct ?(a) 1 and 2(b) 2 only

Page 23: Que

(c) 2 and 3(d) 3 onlyAns: b28. Consider the following assumptions :1. A country under foreign domination cannot indulge in spiritual pursuit.2. Poverty is an  impediment in the spiritual pursuit.3. Subject peoples may become other-wordly.With reference to the passage, which of the above assumptions is/are valid ?(a) 1 and 2(b) 2 only(c) 2 and 3(d) 3 onlyAns: c29. The passage thematically centres on(a) the state of mind of oppressed people (b) starvation and misery (c) the growth of civilization (d) body, mind and spirit of people in generalAns: a30. According to the passage, the torture of the mind and spirit is caused

(a) by the impact of foreign domination.(b) by the desire to escape from foreign domination and find consolation III visions of past greatness.(c) due to lack of equilibrium between an external life and an internal life.(d) due to one's inability to be either revolutionary or other-worldly.Ans: cDirections for the following 3 (Three) items:Read the passage given below, study the graph that follows and answer the three items given belwo the figure.During a party, a person was exposed to cotaminated water. A few days later, he developed fever and loose motions. He suffered for some days before going to a doctor for treatment. On starting the treatment, he soon became better and recovered completely a few days later. The following graph shows different phases of the person's disease condition as regions A, B, C, D and E of the curve.31. Which region/regions of the curve correspond/corresponds to incubation phase of the infection ?(a) A only(b) B only(c) Band C(d) No part of the curve indicates the incubation phaseAns: a32. Which region of the curve indicates that the person began showing the symptoms of infection ?(a) A(b) B(c) C(d) DAns: b33. Which region of the curve indicates that the treatment yielded effective relief?(a) C(b) D(c) E(d) The curve does not indicate the treatmentAns: c34. There are four routes to travel from city A to city B and six routes from city B to city C. How many routes are possible to travel from the city A to city C ?(a) 24(b) 12

Page 24: Que

(c) 10(d) 8Ans: a35. A contract on construction job specifies a penalty for delay in completion of the work beyond a certain date IS as follows : Rs. 200 for the first day,  Rs. 250 for the second day, Rs. 300 for the third day etc., the penalty for each succeeding day being 50 more than that of the preceding day. How much penalty should the contractor pay if he delays the work by 10 days ?(a) Rs. 4950(b) Rs. 4250(c) Rs. 3600(d) Rs. 650Ans: b36. Consider the figure given below and answer the items that follows:

In the figure shown above, OP 1 and OP 2 are two plane mirrors kept perpendicular to each other. S is the direction of a beam of light falIing on the mirror OP 1. Thedirection of the reflected beam of light from the mirror OP 2 will be(a) Perpendicular to the direction S.(b) At 45° to the direction S.(c) Opposite and parallel to the direction S.(d) At 60° to the direction S.Ans: c37. Consider the following tlgure and answer the item that follows:

What is the minimum number of different colours required to paint the figure given above such that no two adjacent regions have the same colour ?(a) 3(b) 4(c) 5(d) 6

Page 25: Que

Ans: a38. Consider the following fogure and answer the items that follows:

A square is divided into four rectangles as shown above. The lengths of the sides of rectangles are natural n umbers. The areas of two rectangles are indicated inthe figure. What is the length of each side of the square ?(a) 10(b) 11(c) 15(d) Cannot be determined as the given data are InsufficientAns: b39.  A person has only Rs. 1 and Rs. 2 coins with her. If the total number of coins that she has is 50 and the amount of money with her is Rs. 75, then the number of Rs. 1 and Rs. 2 coins are, respectively(a) 15 and 35(b) 35 and 15(e) 30 and 20(d) 25 and 25Ans: d40. Three persons start walking together and their steps measure 40 cm, 42 cm and 45 cm respectively. What is the minimum distance each should walk so that each can cover the same distance in complete steps ?(a) 25 m 20 cm(b) 50 m 40 cm(c) 75 m 60 cm(d) 100 m 80 cmAns: a41. If a bus travels 160 km in 4 hours and a train travels 320 km in 5 hours at uniform speeds, then what is the ratio of the distances travelled by them in one hour ?(a) 8 : 5(b) 5 : 8(c) 4 : 5(d) 1 : 2Ans: b42. There are 100 students in a particular class. 60% students play cricket, 30% student play football and 10% student play both the games. What is the number of students who play neither cricket nor football ?(a) 25(b) 20(c) 18(d) 15Ans: b43. A village having a population of 4000 requires 150 liters of water per head per day. It has a tank measuring 20 m x 15 m x 6 m. The water of this tank will last for(a) 2 days(b) 3 days(c) 4 days(d) 5 daysAns: bDirections for the following 4 (four) items:Read the follouing passage and answer the items that follow. Your answers to these items should be based on the passage only.

PassageA species that exerts. an influence out of proportion to its abundance in an ecosystem is called a

Page 26: Que

keystone species. The keystone species may influence both the species richness of communities and the flow of energy and materials through ecosystems. The sea star Pisaster the flow of energy and materials through ecosystems. The sea star Pisaster ochraceus, which lives in rocky intertidal ecosystems on the Pacific coast of North America, is also an example of a keystone species. Its preferred prey is the mussel Mytilus californianus . In the absence of sea- stars, these mussels crowd out other competitors in a broad belt of the intertidal zone. By consuming mussels, sea star creates bare spaces that are taken over by a variety of other species. A study at the University of washington demonstrated the influence of Pisaster on species richness by removing sea stars from selected parts of the intertidal zone repeatedly over a period of five years. Two major changes occured in the areas from which sea stars were removed. First, the lower edge of the 46. mussel bed extended farther down into the intertidal zone, showing that sea stars are able to eliminate mussels completely where they are covered with water most of the time. Second, and more dramatically, 28 species of animals and algae disappeared from the sea star removal zone. Eventually only Mytilus, the dominant competitor, occupied the entire substratum. Through its effect on competitive relationships, predation by Pisaster largely determines which species live in these rocky intertidal ecosystems.44. What is the crux of the passage ?(a) Sea star has a preferred prey.(b) A preferred prey determines the survival of a keystone species.(c) Keystone species ensures species diversity.(d) Sea star is the only keystone species on the Pacific coast of North America.Ans: c45. With reference to the passage, consider the following statements :1. Mussels-are generally the dominant species in intertidal ecosystems.2. The survival of sea stars is generally determined by the abundance of mussels.which of the statements given above is /are correct?(a) 1 only(b) 2 ony(c) Both 1 and 2(d) Neither 1 nor 2Ans: d46. Which of the following is/are implied by the passage ?1. Mussels are always hard competitors for sea stars.2. Sea stars of the Pacific coast have reached the climax of their evolution.3. Sea stars constitute an important component in the energy flow in intertidal ecosystem.Which of the statements given above is/are correct?(a) 1 and 2(b) 2 only(c) 1 and 3(d) 3 onlyAns: d47. Consider the following assumptions:1. The food chains/food web in an influenced ecosystem are keystone species.2. The presence of keystone species is a specific characteristic of aquatic ecosystems.3. If the keystone species is completely removed from an ecosystem, it will lead to the collapse of the ecosystem.With reference to the passage, which of the above assumptions is/are valid?(a) 1 only(b) 2 and 3 only(c) 1 nad 3 only(d) 1, 2 and 3Ans: a48. Consider the following argument:"In, order to be a teacher one must graduate from college. All poets are poor. Some Mathematicians are poets. No college graduate is poor."Which one of the following is not a valid conclusion regarding the above argument?(a) Some Mathematicians are not teachers.(b) Some teachers are not Mathematicians.(c) Teachers are not poor.(d) Poets are not teachers.Ans: b49. A student on her first 3 tests receive. 4 an average score of N points. If she exceeds her previous average score b. 20 points on her fourth test, then what is the average score for the firs 4 tests ?(a) N + 20(b) N + 10

Page 27: Que

(c) N + 4(d) N + 5Ans: d50. In a group of persons, 70% of the persons are male and 30% of the persons are married. If two sevenths of males are married, what fraction of the females is single ?(a) 2/7(b) 1/3(c) 3/7(d) 2/3Ans: d51. The houses of A and B face each other on a road going north-south, A's being on the western side. A comes out of his house, turns left, travels 5 km, turnsright, travels 5 km to the front of D's house. B does exactly the same and reaches the fron t of C's house. In this context, which one of the following statements is correct ?(a) C and D live on the same street.(b) C's house faces south.(c) The houses of C than 20 km apart.d) None of the aboveAns: cDirections for the following 5 (five) items:Read the following passage and answer the items that follow. Your answers to these items should-be based on the passage only.

PassageNow India's children have a right to receive at least eight years of education, the gnawing question is whether' it will remain 'on paper' or 'become a reality. One hardly needs a reminder that this right is different from the others enshrined in the Constitution, that the beneficiary - a six year old child cannot demand it, nor can she or hefight a legal battle when the right is denied or violated. In all cases, it is the adult society which must act on behalf of the child. In another peculiarity, where a child's right to education is denied, no compensation offered later can be adequate or relevant. This is so because childhood does not last. if a legal battle fought on behalf of a child is eventually won, it may be of little use to the boy or girl because the opportunity missed at school during childhood cannot serve the same purpose later in life. This may be painfully true for girls because our society permits them only a short childhood, if at all. The Right to Education (RTE) has become law at a point in India's history when the ghastly practice of female infanticide has resurfaced in the form of foeticide. This is "symptomatic of a deeper turmoil" in society which compounding the traditional obstacles to the education of girls. "Tenacious prejudice against the intellectual potential of girls runs across our cultural diversity and the system of education has not been able to address it.52. With reference to the passage, consider the following statements :1. When children are denied education, adult society does not act on behalf of them.2. Right to Education as a law cannot be enforced in the country.Which of the statements given above is/are correct ?(a) 1 only(b) 2 only(c) Both 1 and 2(d) Neither 1 nor 2Ans: d53. According to the passage, what could be 55 the traditional obstacles to the education of girls ?1.  Inability of parents to fight a legal battle when the Right to Education  is denied to their children.2. The traditional way of thinking about girl's role in society.3. The prejudice against the intellectual potential of girls.4. Improper system of education.Select the correct answer from the codes given below :(a) 1 and 2 only(b) 2, 3 and 4 only(c) 1, 3 and 4 only(d) 1, 2, 3 and 4Ans: b54. On the basis of the passage, consider the following statements:1. Right to Education is a legal right and not a fundamental right.2. For realising the goal of universal education, the education system in the country must be made identical to that of developed countries.Which of the statements given above is/are correct ?(a) 1 only

Page 28: Que

(b) 2 only(c) Both 1 and 2(d) Neither 1 nor 2Ans: d55. Which one of the following statements conveys the key message of the passage ?(a) India has declared that education is compulsory for its children.(b) Adult society is not keen on implementing the Right     to Education.(c) The Right to Education, particularly of a girl child, needs to be safeguarded. (d) The system of education should be address the issue of right to education.Ans: c56. Which one of the following statements conveys the inference of the passage ?(a) The society has a tenacious prejudice against the intellectual potential of girls.(b) Adults cannot be relied upon to fight on behalf of children for their Right to Education.(c) The legal fight to get education for children is often protracted and prohibitive.(d) There is no sufficient substitute for education received in childhood.Ans: dRead the following paesage and answer (three) items that follow:A, B, C, D and E are members of the same family. There are two fathers, two sons, two wives, three males and two females. The teacher was the, wife of a lawyer who was the son of a doctor. E is not male, neither also a wife of a professional. C is the youngest person in the family and D is the eldest. B is a male.57. How is D realated to E ?(a) Husband(b) Son(c) Father(d) WifeAns: a58. Who are the females in the group ?(a) C and E(b) C and D(c) E and A(d) D and EAns: c59. Whose wife is the teacher?(a) C(b) D(c) A(d) BAns: dRead the following passage and ans the 3 (three) items that follow:In a survey regarding a proposal measure to be introduced, 2878 person took part of which 1652 were males. 12 persons voted against the proposal which 796 were males. 1425 persons vote for the proposal. 196 females wet undecided.60. How many females voted for the proposal ?(a) 430(b) 600(c) 624(d) 640Ans: b61. How many males were undecided ?(a) 31(b) 227(c) 426(d) 581Ans: a62. How many females were not in favour the proposal ?(a) 430(b) 496(c) 586(d) 1226Ans: a63. In a queue, Mr. X is fourteenth from the front and Mr. Y is seventeenth from the end, while Mr. Z is exactly in between Mr. X and Mr. Y. If Mr. X is ahead Mr. Y and there are 48 persons in the queue, how many persons are then between Mr. X and Mr. Z ?

Page 29: Que

(a) 6(b) 7(c) 8(d) 9Ans: cDirections for the following 9 (nine) items:The following nine items (Questions 64 to 72) are based on three passages in English to test the comprehension of English language and therefore these items do not have Hindi version. Read each passage and answer the items that follow.

Passage-IHe walked several miles that day but could not get anything to eat or drink except some dry bread and some water, which he got from cottagers and farmers. As night fell, he slept under a haystack lying in a meadow. He felt frightened at first, for the wind blew awfully over the empty fields. He felt cold and hungry, and was feeling more lonely than he had ever felt before. He however, soon fell asleep, being much tired with his long walk. When he got up next day, he was feeling terribly hungry so he purchased a loaf of bread with a few coins that he had.64. When the night fell, he slept(a) in the open field(b) under a pile of dry grass(c) in a farmer's cottage(d) under a treeAns: b65. He soon fell asleep because(a) he was exhausted(b) he was all alone(c) he had not slept for days(d) he was very frightenedAns: a66. With reference to the passage, consider the following statements :1. He was walking through the countryside,2. The cottagers and farmers gave his enough food so that he could sleep at night without feeling hungry.Which of the statements given above is/are correct ?(a) 1 only(b) 2 only(c) Both 1 and 2(d) Neither 1 nor 2Ans: a

Passage - III opened the bag and packed the boots in ; and then , just as I was going to close it, a horrible idea occured to meet Had I packed my toothbrush ? I don't know how it is, but I never do know whether I've packed my toothbrush. My toothbrush is a thing that haunts me when I'm travelling, an makes my life a misery, I dream that haven't packed it, and wake up in a col perspiration, and get out of bed and hur for it. And, in the morning, I pack it before I have used it, and it is always the last thing I turn out of the bag; and then repack and forget it, and have to rug upstairs for it at the last moment an carry it to the railway station, wrapped u in my pocket-handkerchief.67. When he was going to close the bag, the idea that occurred to him was(a) unpleasant(b) sad(c) fantastic(d) amusingAns: a68. What makes his life miserable whenever he undertakes travelling?(a) Going to railway station(b) Forgetting the toothbrush(c) Packing his bag(d) Bad dreamsAns: b69. His toothbrush is finally(a) in his bag(b) in his bed(c) in his handkerchief

Page 30: Que

(d) lostAns: c

Passage-IIIIn spring, polar bear mothers emerge from dens with three month old cubs. The mother bear has fasted for as long as eight months but that does not stop the youngfrom demanding full access to her remaining reserves. If there are triplets, the most persistent stands to gain an extra meal and it may have the meal atthe expense of others. The smallest of the litter forfeits many meals to stronger siblings. Females are protective of their cubs but tend to ignore family rivalry overfood. In 21 years of photographing polar bears, I've only once seen the smallest of triplets survive till autumn.70. Female polar bears give birth during(a) spnng(b) summer(c) autumn(d) winterAns: d71. Mother bear(a) takes sides over cubs(b) lets the cubs fend for themselves(c) feeds only their favourites(d) see that all cubs get an equal shareAns: b72. With reference to the passage, the following assumptions have been made:1. Polar bears fast as long as eight months due to non-availability prey.2. Polar bears always give birth to triplets.Which of the assumptions given above is/are valid?(a) 1 only(b) 2 only(c) Both 1 and 2(d) Neither 1 nor 2Ans: d

Directions for the following 8 (eight) items:Given below are eight items. Each item describes a situation and is followed by four possible responses. Indicate the response you find most appropriate. Choose only one response for each item. The responses will be evaluated based on the level of appropriatehess for the given situation.

Please attempt all the items. There is no penalty for wrong answers for these eight items.

73. You have been asked to give an explanation for not attending an important official meeting. Your immediate boss who has not informed you about the meeting is now putting pressure on you not to place an allegation against him / her. You would(a) send a written reply explaining the fact.(b) seek an appointment with the top boss to explain the situation.(c) admit your fault to save the situation.(d) put the responsibility on the coordinator of the meeting for not informing.Ans: a74. A local thug (bad element) has started illegal construction on your vacant plot. He has refused your request to vacate and threatened you of dire consequences incase you do not sell the property at a cheap price to him. You would(a) sell the property at a cheap price to him.(b) go to the police for necessary action.(c) ask for help from your neighbours.(d) negotiate with the goon to get a higher price.Ans: b75. You have to accomplish a very important task for your headquarters within the next two days. Suddenly you meet with an accident. Your office insists that you complete the task. You would(a) ask for an extension of deadline.(b) inform Headquarters of your inability to finish on time.(c) Suggest alternate person to headquarters who may do the needful.(d) stay away till you recover.Ans: b

Page 31: Que

76. You are an officer-in-charge for providing basic medical facilities to the survivors of an earthquake affected area. Despite your best possible effort, people put allegations against you for making money out of the funds given for relief. You would(a) let an enquiry be set up to look into the matter.(b) ask your senior to appoint some other person in your place.(c) not pay attention to allegations.(d) stop undertaking any initiative till the matter is resolved.Ans: a77. You have been made responsible to hire boats at a short notice to be used for an area under flood. On seeing the price mentioned by the boat owners you found that the lowest price was approximately three times more than the approved rate of the Government. You would(a) reject the proposal and call for a fresh price.(b) accept the lowest price.(c)  refer the matter to the Government and wait.(d) threaten the boat owners about a possible cancellation of the licence.Ans: b78. You are the officer-in-charge of a village administering distribution of vaccine in an isolated epidemic hit village, and you are left with only one vaccine. There is a requirement of that vaccine from the GramPradhan and also a poor villager. You are being pressurised by the Gram Pradhan to issue the vaccine to him. You would(a) initiate the procedure to expedite the next supply without issuing the vaccine to either.(b) arrange vaccine for the poor villager from the distributor of another area.(c) ask both to approach a doctor and get an input about the urgency.(d) arrange vaccine for the Gram Pradhan from the distributor of another area.Ans: c79. You have taken up a project to create night-shelters for homeless people during the winter season. Within a week of establishing the shelters, you have received complaints from the residents of the area about the increase in theft cases with a demand to remove the shelters. You would(a) ask them to lodge a written complaint in the police station.(b) assure residents of an enquiry into the matter.(c) ask residents to consider the humanitarian effort made.(d) continue with the project and ignore their complaint.Ans: b80. You, as an administrative authority, have been approached, by the daughter-in-law of an influential person regarding harassment by her in-laws on account of insufficient dowry. Her parents are not able to approach you because of social pressures. You would(a) call the in-laws for an explanation.(b) counsel the lady to adjust, given such a circumstance.            (c) take action after her parents approach you.                (d) ask her to lodge complaint with the police.Ans: d

BPSC Combined (Pre) Competitive Examination 2011, Solved Question Paper

BPSC Combined (Pre) Competitive Examination 2011, Solved Question PaperSub Date: 4/17/2011

The Bihar Public Service Commission (BPSC) conducted 53rd to 55th Common Combined (Pre) Competitive Examination 2011 on April 17, 2011. Here you find its solved question paper. In the Bihar Public Service Commission PT 2011 Exam's Solution, jagranjosh.com has offered you the authentic answers for all questions. Our Expert team has done a comprehensive and exhaustive research work from all the possible authentic sources to produce the solutions of the questions that a competitive exam's student can really rely upon. Learn and judge yourself with jagranjosh.com solved BPSC PT 2011 questions; make your exam preparation strong, broad and dynamic.1. In which country is 'Takla Makan' desert situated?(A) Kazakhstan(B) Turkmenistan(C) Uzbekistan

Page 32: Que

(D) China

Answer: D

2. Which strait connects Red Sea and Indian Ocean?(A) Bab-el-Mandeb(B) Hormuz(C) Bosporus(D) Malacca

Answer: A

3. Where is 'Ninety East Ridge' situated?(A) Pacific Ocean(B) Indian Ocean(C) Atlantic Ocean(D) Arctic Ocean

Answer: B

4. Which of the following is not a capital city?(A) Canberra(B) Sydney(C) Wellington(D) Riyadh

Answer: B

5. The highest coal-producing country in the world is(A) India(B) USA(C) China(D) Russia

Answer: C

6. Folding is the result of(A) epeirogenetic force(8) Coriolis force(C) orogenetic force(D) exogenetic force

Answer: C

7. Amritsar and Shimla are almost on the same latitude, but their climate difference is due to(A) the difference in their altitudes(B) their distance from sea(C) snowfall in Shimla(D) pollution in Amritsar

Answer: A

8. Coal mines in Jharkhand are located at(A) Jharia(B) Jamshedpur(C) Ranchi(D) Lohardaga

Answer: A

9. Major source of oceanic salinity is(A) rivers(B) land(C) wind

Page 33: Que

(D) ash from volcanoes

10. The large States of India in the order of area are(A) Rajasthan, Madhya Pradesh, Maharashtra(B) Madhya Pradesh, Rajasthan, Maharashtra(C) Maharashtra, Rajasthan, Madhya Pradesh(D) Madhya Pradesh, Maharashtra, Rajasthan

Answer: A

11. Bihar is the -- largest State of India according to Census 2001.(A) second(B) third(C) fourth(D) fifth

Answer: B

12. The highest wheat-producing State of India is(A) Haryana(B) Punjab(C) Bihar(D) Uttar Pradesh

Answer: D

13. TISCO plant is located near(A) Patna(B) Darbhanga(C) Dhanbad(D) Tatanagar

Answer: D

14. Which States in India are the largest producers of sugarcane?(A) Bihar and Uttar Pradesh(B) Uttar Pradesh and Rajasthan(C) Andhra Pradesh and Jammu and Kashmir(D) Punjab and Himachal Pradesh

Answer: A

15. Which of the following States has the largest number of people belonging to Scheduled Caste?(A) Bihar(B) West Bengal(C) Uttar Pradesh(D) Punja

Answer: C

16. According to 2001 Census, what is the total population of Bihar?(A) 82998509(B) 76210007(C) 96878627(D) 80176197

Answer: A

17. The East Central Railway zone headquarters is located at(A) Patna(B) Hajipur(C) M uzaffarpur(D) Katihar

Page 34: Que

Answer: B

18. Where is Industries Commissioner Office of Bihar situated?(A) Patna(B) Gaya(C) Hajipur(D) Muzaffarpur

Answer: A

19. North Bihar is famous for(A) agricultural prosperity(B) heavy industries(C) flood(D) famine

Answer: C

20. Which one is not located on National Highway-2 (NH-2)?(A) Aurangabad(B) Sasaram(C) Mohania(D) Patna

Answer: D

21. Congress passed the 'Swaraj' resolution in the year 1905. The purpose of the resolution was(A) right to make a constitution for themselves but it was not done(B) to secure self-rule(C) responsible government(D) self-government

Answer: B

22. The Provincial were constituted Act of Governments under the(A) 1935(B) 1932(C) 1936(D) 1947

Answer: A

23. The Indian comprises of Constitution(A) 395 Articles, 22 Parts and 12 Schedules(B) 371 Articles, 21 Parts and 11 Schedules(C) 372 Articles, 20 Parts and 7 Schedules(D) 381 Articles, 23 Parts and 8 Schedules

Answer: A

24. The Rajya Sabha consists of(A) 280 members of which 20 members are nominated by the President of India(B) 275 members of which 18 members are nominated by the President of India(C) 250 members of which 12 members are nominated by the President of India(D) 252 members of which 12 members are nominated by the President of India

Answer: C

25. January 26 was selected as the date for the inauguration of the Constitution, because(A) the Congress had observed it as the Independence Day in 1930(B) on that day the Quit India Movement was started in 1942(C) it was considered to be an auspicious day(D) None of the above

Page 35: Que

Answer: A

26. The Public Accounts Committee submits its report to(A) the Comptroller and Auditor General(B) the Speaker of the Lok Sabha(C) the Minister of Parliamentary Affairs(D) the President of India

Answer: B

27. Of the following statements, which one is not correct?(A) The Rajya Sabha is powerless in money matter(B) Money Bills originate in the Rajya Sabha(C) The Rajya Sabha has to pass Bills within 14 days after they are passed by the Lok Sabha(D) The Rajya Sabha may pass or return the Money Bill with some recommendations to the Lok Sabha

Answer: B

28. Which of the following rights conferred by the Constitution of India is also available to non-citizens?(A) Right to constitutional remedies(B) Freedom to speech(C) Freedom to move and settle in any part of the country(D) Freedom to acquire property

Answer: A

29. Who is the head of the National Defence Committee?(A) Home Minister(B) Prime Minister(C) President(D) Vice-President

30. 'National Emergency' may be declared by the President of India in line with(A) Article 352(B) Article 370(C) Article 371(D) Article 395

Answer: C

31. Of the following statements, which one is not correct?(A) Supreme Court was constituted in 1950(B) Supreme Court is the highest court of appeal in the country(C) Supreme Court can hear from any High Court/Tribunals except from Court-martial(D) Supreme Court can hear from any High Court/ Tribunals as well as from Court-martial

Answer: D

32. Who is the constitutional head of the State Governments?(A) Chief Minister(B) Governor(C) Speaker(D) High Court Judge

Answer: B

33. Commercial sources of energy purely consist of(A) power, coal, oil, gas, hydro-electricity and uranium(B) coal, oil, firewood, vegetable waste and agricultural waste(C) power, coal, animal dung and firewood(D) coal, gas, oil and firewood

Answer: A

Page 36: Que

34. In India, per capita consumption of energy in 1994 was(A) 300 kg of oil equivalent(B) 360 kg of oil equivalent(C) 243 kg of oil equivalent(D) 343 kg of oil equivalent

Answer: A

35. When did the first oil crisis/ energy crisis occur in India?(A) During 1950's and 1960's(B) During 1930's and 1940's(C) During 1990's and 2000's(D) During 1970's and 1980's

Answer: D

36. Consider the following reasons for financial sickness of State Electricity Boards (SEBs) in India:I. Sale of power to agricultural and domestic consumers below the cost of productionII. Transmission and distribution losses are very highIII. Lack of commercial autonomy for SEBsIV. State Governments have implemented social subsidy policies through SEBs

37. Which of the above are correct?(A) I, II and III(B) I, II, III and IV(C) I, III and IV(D) II, III and IV

37. Which type of economy does India have?(A) Socialist(B) Gandhian(C) Mixed(D) Free

Answer: C

38. When was 'Speed Post Service' launched by the Indian Postal Department as competition to the 'Courier Service'?(A) 1988(B) 1987(C) 1989(D) 1986

Answer: D

39. We witnessed the first telephones in India in which year?(A) 1951(B) 1981(C) 1851(D) 1861

Answer: D

40. Consider the following problems being faced by the Indian Coal Industry :I. Poor quali ty of coal and bottlenecks m the coal movementII. Low utilisation capacity of washeriesIII. Growing dependence on the import of coking coalIV. Administered pricesWhich of the above are correct?(A) II, III and IV(B) I, II, III and IV(C) I, III and IV(D) I, II and III

Page 37: Que

Answer: B

41. Name the committee which was set up by the NDC in 1957 for the reorganisation of CDP and NES, which suggested a three tier system of rural local government(A) Balwant Rai Mehta Committee(B) Ashok Mehta Committee(C) Maharashtra Committee on Democratic Decentralisation(D) Rural-Urban Relationship Committee

Answer: A

42. Ashok Mehta recommended(A) three-tier government of Panchayati Raj Committee(B) two-tier government of Panchayati Raj(C) single-tier government of Panchayati Raj(D) multiple-tier government of Panchayati Raj

Answer: B

43. Consider the following about the 'Rolling Plan':I. A plan for the current year which includes the annual budgetII. A plan for a fixed number of years, say 3, 4 or 5III. It is revised every year as per requirements of the economyIV. A perspective plan for 10, 15 or 20 yearsWhich of the above are correct?(A) I and II(B) I and III(C) II and III(D) I, II, III and IV

Answer: B

44. In which year was the 'Planning Commission' set up in India?(A) 1950(B) 1947(C) 1948(D) 1951

Answer: A

45. Patna International Airport is directly connected with(A) Kathmandu (Nepal), New Delhi, Kolkata, Mumbai, Lucknow, Varanasi and Ranchi(B) Bengaluru, Hyderabad and Dhaka(C) Islamabad, Dhaka and Bengaluru(D) Washington, Dhaka and Chennai

Answer: A

46. In Bihar, air services are provided by(A) only Indian Airlines(B) only Sahara Airlines(C) only Royal Nepal Airlines(D) Indian Airlines, Sahara Airlines and Royal Nepal Airlines

Answer: D

47. For Bihar, in 1993-94, the Planning Commission estimated that percentage of below poverty line (BPL) was(A) 55(B) 65(C) 45(D) 35

48. In Bihar, though 'Zamindari' was statutorily abolished In 1952, the social base of land control

Page 38: Que

remained in the grip of(A) Middle Caste Hindus(B) Scheduled Caste Hindus(C) Dominant Caste Hindus(D) Scheduled Tribe Hindus

Answer: C

49. Bihar State (before separation) had how many 'Industrial Area Development Authorities'?(A) Six, viz., Adityapur, Bokaro, Patna, Ranchi, Darbhanga and Muzaffarpur(B) Five, viz., Adityapur, Bokaro, Darbhanga, Muzaffarpur and Patna(c) Seven, viz., Adityapur, Darbhanga, Ranchi and Bokaro, Patna, Muzaffarpur, Bhojpur(d) Four, viz., Adityapur, Bhojpur and Muzaffarpur, Khagaria

50. In Bihar, name the scheme which aims at building up infrastructural facilities of high standard, required for establishment of 'Export oriented Units'(A) Integrated Infrastructural Development (IID)(B) Export Promotion Industrial Park (EPIP)(C) Concept of Build-Operate- Transfer (BOT)(D) Software/Hardware Technology Park (STP /HTP)

51. "Swaraj is my birthright and I shall have it." Who said?(A) M. K. Gandhi(B) Jawaharlal Nehru(C) Bal Gangadhar Tilak(D) Bhagat Singh

Answer: C

52. The Madras Mahajan Sabha was established in the year(A) 1880(B) 1881(C) 1882(D) 1883

Answer: D

53. How many representatives took part in the First Session of the Indian National Congress?(A) 52(B) 62(C) 72(D) 82

Answer: C

54. The first Indian Governor General of Independent India was(A) Raja Gopalachari(B) Surinder Nath(C) Rajendra Prasad(D) B. R. Ambedkar

Answer: A

55. When was the National Council of Education established?(A) 15th August, 1903(B) 15th August, 1904(C) 15th August, 1905(D) 15th August, 1906

Answer: D

56. The Non-Cooperation Movement was started in(A) 1918(B) 1920

Page 39: Que

(C) 1921(D) 1922

Answer: B

57. The Montagu-Chelmsford Report formed the basis of(A) the Indian Independence Act, 1947(B) the Indian Councils Act, 1909(C) the Government of India Act, 1919(D) the Government of India Act, 1935

Answer: C

58. Who is popularly known as 'Sher-e-Punjab,?(A) Rajguru(B) Bhagat Singh(C) Lala Lajpat Rai(D) Udham Singh

Answer: C

59. Gandhi-Irwin Pact took place in(A) 1930(B) 1931(C) 1932(D) 1933

Answer: B

60. Which Round Table Conference held in 1932?(A) First(B) Second(C) Third(D) Fourth

Answer: C

61. Who is associated with 'Abhinav Bharat'?(A) V. D. Savarkar(B) C. R. Das(C) B. G. Tilak(D) S. C. Bose

Answer: A

62. 'Do or Die' is associated with the following movement(A) Dandi(B) Non-Cooperation(C) Khilafat(D) Quit India

Answer: D

63. With whose efforts was Second Bihar Provincial Congress Session held?(A) G. K. Gokhale(B) Shri Deepnarayan Singh(C) Shri Krishna(D) Surendranath Banerjee

Answer: B

64. At which place was the 27th Session of the Indian National Congress held?(A) Bhagalpur(B) Patna

Page 40: Que

(C) Ranchi(D) Bankipur

65. The 16th Session of Bihari Students' Conference at Hazaribag took place in(A) 1921(B) 1922(C) 1923(D) 1924

Answer: A

66. The editor of 'Bihari', a leading newspaper of Bihar, was(A) Babu G. Prasad(B) Babu Ram Prasad(C) Babu Gopal Prasad(D) Babu Maheshwar Prasad

Answer: D

67. Raj Kumar Shukla was resident of the village(A) Murli Bharhawa(B) Murli Bheet(C) Murli Dheer(D) Murli Kher

Answer: A

68. Shri Nivaranchandra Dasgupta belonged to(A) Patna(B) Purulia(C) Hazipur(D) Meenapur

Answer: B

69. Kunwar Singh was the King of(A) Hamirpur(B) Dheerpur(C) Jagdishpur(D) Rampur

Answer:C

70. Lord Sinha had resigned in 1921 from the post of Governor of(A) Bihar-Orissa(B) Bengal(C) Punjab(D) Madras

Answer: A

71. The value of x - [y - {z - (x - y - z)}j is(A) x+ y+z(B) x - y-z(C) 1(D) °

Answer: C

72. If the function f : I --? R, f(x) = log x then the value of f(x) + f(y) is(A) f(xy)(B) f(x+ y)(C) f (x/y)(d) f(y/x)

Page 41: Que

Answer: C

73. The median of 2, 4, 6, 8, 10, 12, 14, 16 is(A) 8(B) 9(C)10(D)11

Answer: B

74. The inverse of the matrix is(B)(C)(D)

Answer: C

75. The length of the minute hand of a clock is 12 cm. Then the area swept by the minute hand in one minute will be(A) 22·12 cm2(B) 23·10 cm2(C) 24·12 cm2(D) None of the above

Answer: D

76. A player completes 2200-metre race in any circular path with 7 revolutions. Then the radius of the circular path is ( 1t = ~2)(A) 30 metres(B) 40 metres(C) 50 metres(D) 60 metres

Answer: C

77. The coordinates of the mid- point on joining the points P (4, 6) and Q (- 4, 8) will be(A) (2, 7)(B) (7, 2)(C) (7, 0)(D) (0, 7)

Answer: D

78. The solution of the differential equationd2 d-----.l:L - 3-.1{ + 2y = e5xdx2 dxisA) y = C eX + C e2x + ~e5x1 2 12B) Y = C e-x + C e2x + ~e5X1 2 12C) y = C eX + C e-2x + ~ e5x1 2 12

79. In triangle PQR if L.P = 1200 and PQ = PR, then L. Q and L.R will be respectively(A) 60°, 30°

(B) 30°, 40°

(C) 30°, 30°

(D) 20°, 40°

Page 42: Que

Answer: C

80. The value of is

lim X→0

(A) 0(B) loge ~ b(C) log - ea(D) a- b

Answer: B

81. The visible range of solar radiation is(A) 100-400 nm (B) 400-700 nm (C) 740-10000 nm (D) None of the above

Answer: B

82. Plants which grow on saline soils are(A) xerophytes(B) hydrophytes

(C) halophytes(D) succulents

Answer: C

83. Environment is a composite state of(A) biotic factors(B) physiographic factors (C) abiotic factors(D) All of the above

Answer: D

84. Water is conducted in vascular plants by(A) phloem tissue(B) parenchyma tissue (C) meristems(D) xylem tissue

Answer: C

85. Which component of plants receives stimulus for flowering?

(A) Stems

(B) Branches

(C) Leaves

(D) Roots

86. Minerals are(A) liquids(B) inorganic solids(C) gases(D) All of the above

Answer: C

87. Development of the natural systems is described as(A)~nction of the systems(B) evolution of the systems(C) self-sustained process of the systems(D) None of the above

Page 43: Que

Answer: C

88. Genetics deals with

(A) Mendel's laws(B) organic evolution(C) DNA structure(D) heredity and variations

Answer: C

89. Mendel's principles of inheri- tance are based on(A) vegetative reproduction (B) asexual reproduction (C) sexual reproduction(D) All of the above

Answer: C

90. Embryo is found in

(A) flowers(B) leaves(C) seeds(D) buds

Answer: C

91. What soil particles are present in loamy soils?(A) Sand particles(B) Clay particles (C) Silt particles(D) All types of particles

Answer: D

92. A single type of atom is found in

(A) compounds of minerals(B) mixture of minerals(C) native elements(D) None of the above

Answer: C

93. Soil water available to plants is maximum in(A) clayey soil (8) silty soil (C) sandy soil (D) loamy soil

Answer: C

94. Capillaries are most effective in (A) clayey soil(8) silty soil(C) sandy soil(D) loamy soil

Answer: C

95. Photosynthesis occurs in

(A) nucleus

(b) mitochondria

(C) chloroplast

(D) peroxisome

Page 44: Que

Answer: C

96. Continents have drifted apart because of(A) volcanic eruptions (8) tectonic activities(C) folding and faulting of rocks (D) All of the above

Answer: C

97. Which of the following do not belong to solar system?(A) Asteroids(8) Comets(C) Planets(D) Nebulae

Answer: C

98. Table salt (NaCI) is a product of(A) weak acid and weak base(B) strong acid and strong base(C) weak acid and strong base(D) strong acid and weak base

99. Sexual reproduction causes genetic variation because of(A) blending of genes(B) chromosomal changes(C) Shuffling of genes(D) All of the above

Answer: D

100. Average salinity of water of Arabian Sea is(A) 25 ppt (8) 35 ppt (C) 45 ppt (D) 55 ppt

Answer: C

101. Who among the following is the winner of the Mahatma Gandhi International Award for Peace and Reconciliation, 2009?(A) Yukia Amano(B) Aung San Suu Kyi(C) Bill Gates(D) Hillary Clinton

Answer: B

102. 'Durand Cup' is associated with the game of

(A) football(B) polo(C) cricket(D) hockey

Answer: A

103. For which word, the letter 'G' has been used in '2G Spectrum'?(A) Global(B) Government(C) Generation(D) Google

Answer: C

104. Former Central Minister for Communication and Information Technology, Mr. A. Raja belongs to which of the following parties?(A) Congress

Page 45: Que

(B) BJP(C) Communist Party of India(D) DMK

Answer: D

105. Which one of the following numbered Rajdhani trains covers the longest distance?(A) 12429 Bangalore City Junction(B) 12431 Trivandrum Central(C) 12433 Chennai Central(D) 12435 Dibrugarh Town

Answer: B

106. What is the minimum age of a person whose photographing, fingerprinting and iris mapping will be done for the Census of 2011?

(A) 12 years(B) 15 years(C) 18 years(D) 21 years

Answer: B

107. The Union Cabinet, on August 2010, decided to form institutes like the All India Institute of Medical Sciences (AIIMS) in which six States?(A) Bihar, Chhattisgarh, Madhya Pradesh, Orissa, Rajasthan, Uttarakhand(B) Bihar, Uttar Pradesh, Himachal Pradesh, Haryana, Kerala, Gujarat(C) Bihar, Uttarakhand, Jharkhand, Orissa, Andhra Pradesh, Maharashtra(D) Bihar, Tamil Nadu, Karnataka, Uttar Pradesh, Jammu and Kashmir, Assam

Answer: A

108. Which was reported the richest party according to the 'National Election Watch' on August 2010?(A) BSP(B) BJP(C) Socialist Party(D) Congress

Answer: D

109. Which party-wise performance is correct for Bihar Assembly Elections, 2010 in comparison to the elections of 2005?JDU BJP RJD WP CONG OTHERS(A) +40 +25 -30 -10 -7 -19(B) +25 +34 -20 -5 -3 -11(C) +27 +36 -32 -7 -5 -19(D) + 17 +46 -15 -10 -10 -10

Answer: C

110. Till 2010, which State Governments have provided 50 percent reservation for women in local bodies?(A) Bihar, Uttar Pradesh, Rajasthan(B) Bihar, Madhya Pradesh, Himachal Pradesh(C) Bihar, Madhya Pradesh, Kerala(D) Bihar, Himachal Pradesh, Uttar Pradesh

Answer: B

111. 'AVAHAN', the Indian segment of 'Bill and Melinda Gates Foundation', is engaged in the prevention of(A) dengue(B) polio

Page 46: Que

(C) HIV / AIDS(D) filariasis

Answer: C

112. A recently held underwater meeting has drawn attention world over. The meeting was held by(A) Some Somali pirates to make a new strategy for capturing ships for ransom(B) Some environmental scientists for promoting awareness for saving water(C) some activists environmental awareness for promoting against air pollution(D) The cabinet of a country to highlight the threat of rising sea level for any country

Answer: D

113. Which one of the following cities is not located on the bank of river Ganga?(A) Fatehpur(B) Bhagalpur(C) Uttarkashi(D) Kanpur

Answer: C

114. Which one of the following is a human right as well as a fundamental right under the Constitution of India?(A) Right to Information(B) Right to Work(C) Right to Education(D) Right to Housing

Answer: C

115. The time at Cairo is 2 hours ahead of Greenwich. Hence, it is located at(A) 30° W longitude(B) 30° E longitude(C) 28° E longitude(D) 28° W longitude

Answer: B

116. The headquarters of International Court of Justice is at(A) Hague(B) New York(C) Geneva(D) Paris

Answer: A

117. In the world's best countries' list, compiled by the reputed 'News Week' magazine in 2010, the first position went to Finland. What was the rank of India?

(A)17(B)48(C)66(D)78

Answer: D

118. Which university was ranked first in the list of '2010 Academic Rankings of the World Universities '?(A) Jawaharlal Nehru University, India(B) Cambridge University, Britain(C) Harvard University, USA(D) Massachusetts Institute of Technology, USA

Answer: C

Page 47: Que

119. Which date is known as 'Diabetes Day'?(A) 14th February(B) 14th May(C) 14th September(D) 14th November

Answer: D

120. In which country, Ms. Julia Gillard was made the first woman Prime Minister in the year 2010?(A) Australia(B) Canada(C) Germany(D) Poland

Answer: A

121. Which country's parliament has banned 'Burqa' by passing a law on 14th September, 2010?(A) USA(B) France(C) Italy(D) Holland

Answer: B

122. Mr. Arjun Munda was on September 11, 2010 sworn in as the Chief Minister of Jharkhand. How many MLAs in the 81- member Assembly supported him?(A) 41(C) 50(B) 45(D) 55

Answer: B

123. Mr. P. J. Thomas has been appointed as the Central Vigilance Commissioner on September 7,2010. He is an IAS Officer of the following State's cadre(A) Assam(B) Karnataka(C) Kerala(D) Tamil Nadu

Answer: C

124. What was the maximum amount which has been provided as financial autonomy by the Ministry of Heavy Industries and Public Enterprises of Union Government to the 'Mini Ratna', category I?(A) Rs 250 crore(B) Rs 350 crore(C) Rs 400 crore(D) Rs 500 crore

Answer: D

125. When was the Right to Education added through the amendment in the Constitution of India?(A) 1st April, 2010(B) 1st August, 2010(C) 1st October, 2010(D) 1st December, 2010

Answer: A

126. Match the achievements of India in the XVI Asian Games, 2010 :1. Rank P. 142. Gold Q. 333. Silver R. 064. Bronze S. 17

Page 48: Que

(A.) 1 2 3 4Q R P S(B) 1 2 3 4R P S Q(c.) 1 2 3 4P Q R S(D) 1 2 3 4S R P Q

Answer: C

127. To which institute, Shubha Tole, the first woman awardee of prestigious Shanti Swarup Bhatnagar Prize, 2010 in the field of Science, belongs?(A) Tata Institute of Fundamental Research, Mumbai(B) Indian Institute of Science Education and Research, Pune(C) Indian Institute of Technology, Kanpur(D) Indian Institute of Science"Bengaluru

Answer: A

128. Which Indian was honoured by the 'World Statesman Award, 2010'?(A) Mrs. Sonia Gandhi(B) Atal Bihari Vajpayee(C) Dr. Manmohan Singh(D) Lal Krishna Advani

Answer: C

129. India's Third Research Centre at Antarctica is name as(A) Bharti(B) Swagatam(C) Hindustan(D) Maitri

Answer: A

130. In Commonwealth Games, 2010, India won 101 medals. Out of them, how many medals have been received by team events, women and men respectively?(A) 0, 37, 64(B) 2, 35, 64(C) 1, 37, 63(D) 1, 36, 64

Answer: B

131. Which of the following is a Harappan port?(A) Alexandria(B) Lothal(C) Mahasthangarh(D) Nagapattanam

Answer: B

132. The Gupta emperor who defeated the 'Hunas' was(A) Samudragupta(B) Chandragupta II(C) Skandagupta(D) Ramgupta

Answer: C

133. What is the name of Kalhana's book?(A) Arthashastra(B) Indica

Page 49: Que

(C) Purana(D) Rajtarangini

Answer: D

134. Which inscription tells about the various achievements of Rudradaman I?(A) Junagarh(B) Bhitari(C) Nasik(D) Sanchi

Answer: A

135. Who was the first ruler of the Slave dynasty?(A) Qutubuddin Aibak (B) Iltutmish (C) Razia (D) 'Balban

Answer: A

136. Who was the ruler of Devgiri at the time of Alauddin Khalji's invasion?(A) Prataprudra bev (B) Ramchandra Dev (C) Malik Kafoor (D) Rana Ratan Singh

Answer: B

137. Who of the following Muslim rulers abolished the pilgrimage tax?(A) Bahalol Lodi (B) Sher Shah (C) Humayun (D) Akbar

Answer: D

138. What was 'Dar-ul-Shafa' established by Firoz Tughlaq?(A) An alms house(B) A free hospital(C) A library(D) A guesthouse for pilgrims

Answer: B

139. The 'Permanent Settlement' was made with(A) Zamindars (B) village communities (C) Muqaddamas (D) peasants

Answer: A

140. Who was the founder of 'Prarthana Samaj?(A) Atmaram Panduranga (B) Tilak (C) Annie Besant (D) Rasbihari Ghosh

Answer: A

141. Who was the father of Indian Renaissance?(A) Bal Gangadhar Tilak(B) Dayananda Saraswati(C) Shraddhananda(D) Raja Ram Mohan Roy

Answer: D

142. 'Swaraj Dal' was founded by(A) Tilak and Chittaranjan Das(B) Gandhi and Motilal Nehru(C) Gandhi and Tilak(D) Chittaranjan Das and Motilal Nehru

Answer: D

143. Who founded the 'Theosophical Society'?

Page 50: Que

(A)Madame H, P. Blavatsky(B) Raja Ram Mohan Roy(C) Mahatma Gandhi(D) Swami Vivekananda

Answer: A

144. Which one of the following books was written by Tilak?

(A) India in Transition(B) Gita Rahasya(C) Gokhale-My Political Guru(D) Discovery of India

Answer: B

145. Where was Mahaveer Swami born?(A) Kundagram(B) Pataliputra(C) Magadh(D) Vaishali

Answer: D

146. Where did Mahatma Buddha's 'Mahaparinirvan' take place?(A) Lumbini(B) Bodh Gaya(C) Kushinara(D) Kapilavastu

Answer: C

147. At which place did Mahatma Buddha give his first 'Dharmachakrapravartan '?(A) Lumbini(B) Sarnath(C) Pataliputra(D) Vaishali

Answer: B

148. What was the early capital of Magadh?(A) Pataliputra(B) Vaishali(C) Rajagriha (Girivraja)(D) Champa

Answer: C

149. What was the name of the dynasty of Ajatshatru?(A) Maurya(B) Haryanka(C) Nanda(D) Gupta

Answer: B

150. The Third Buddhist Council was convented at(A) Takshashila(B) Sarnath(C) Bodh Gaya(D) Pataliputra

Answer: D

Page 51: Que

Preparing Current A

IAS Essay : Past Eleven Years Paper

IAS Essay : Past Eleven Years PaperSub Date: 5/4/2011

(I). Independent thinking should be encouraged right form the childhood.

(II). Evaluation of Panchayati Raj System in India from the point of view of eradication of power to people.

(III). Attitude makes, habit makes character and character makes a man.

(IV). Is Autonomy the best answer to combat balkanization?

(V). How has satellite television brought about cultural change in Indian mindsets.

(VI). BPO boom in India

2006

1. Women's Reservation Bill Would Usher in Empowerment for Women in India.2. Protection of Ecology and Environment is Essential for Sustained Economic Development.3. Importance of Indo-U.S. Nuclear Agreement4. "Education for All" Campaign in India: Myth or Reality.5. Globalization Would Finish Small-Scale Industries in India.6. Increasing Computerization Would lead to the Creation of a Dehumanized Society.

2005

1. Justice must reach the poor2. The hand that rocks the cradle3. If women ruled the world4. What is real education?5. Terrorism and world peace6. Food security for sustainable national development

2004

1. India's Role in Promoting ASEAN Co-operation.2. Judicial Activism and Indian Democracy.3. Whither Women's Emancipation?4. Globalizations and Its Impact on Indian Culture.5. The Lure of Space.6. Water Resources Should Be Under the Control of the Central Government.

2003

1. The Masks of New Imperialism.2. How far has democracy in India delivered the goods?3. How should a civil servant conduct himself?4. As civilization advances culture declines.5. There is nothing either good or bad but thinking makes it so.6. Spirituality and Scientific temper.

2002

1. Modern technological education and human values.2. Search for truth can only be a spiritual problem.3. If youth knew, if age could.4. The paths of glory lead but to the grave.5. Privatization of higher education in India.6. Responsibility of media in a democracy.

2001

Page 52: Que

1. What have we gained from our democratic set-up?2. My vision of an ideal world order.3. The march of science and the erosion of human values.4. Irrelevance of the classroom.5. The pursuit of excellence.6. Empowerment alone cannot help our women.

2000

1. Why should we be proud of being Indians?2. The cyberworld: Its charms and challenges.3. The country's need for a better disaster management system.4. Indian culture today: A myth or a reality?5. The implications of globalization for India.6. Modernism and our traditional socio-ethical values.

1999

1. Women empowerment: Challenges and prospects.2. Youth culture today.3. Mass media and cultural invasion.4. Resource management in the Indian context.5. Value-based science and education.6. Reservation, politics and empowerment.

1998

1. The composite culture of India.2. Woman is God's best creation.3.The misinterpretation and misuse of freedom in India.4. India's contribution to world wisdom.5. The language problem in India: Its past, present and prospects.6. The world of the twenty-first century.

1997

1. What we have not learnt during fifty years of Independence.2. Judicial activism.3. Greater political power alone will not improve women's plight.4. True religion cannot be misused.5. The modern doctor and his patients.6. Urbanization is a blessing in disguise.

26. Bureaucracy that is committed to the programmes of the political party in power is called—(A) Depoliticised bureaucracy(B) Semi-politicised bureaucracy(C) Committed bureaucracy(D) Fully politicised bureaucracyAns : (C)

27. The principle of 'span of control' means—(A) An employee should receive orders from one superior only(B) The number of subordinate employees that an administrator can effectively direct(C) The control or supervision of the superior over the subordinate(D) The number of people being controlledAns : (B)

28. Which of the following is not a function of staff agency ?(A) Planning(B) Advising(C) Consultation(D) Achieving goalsAns : (D)

29. Which of the following is not a staff agency in India ?(A) Cabinet Secretariat(B) Cabinet Committees

Page 53: Que

(C) Planning Commission(D) Economic Affairs DepartmentAns : (D)

30. In hierarchy, the term 'scalar' means—(A) Step(B) Ladder(C) Position(D) ProcessAns : (B)

31. The 4Ps theory of departmentalization as advocated by Luther Gulick are—(A) Purpose, process, plan, place(B) Place, person, programme, process(C) Purpose, process, place, programme(D) Purpose, process, person, placeAns : (D)

32. One who tells one's supervisor anything detrimental to an associate is called—(A) A squealer(B) A rate buster(C) A chiseller(D) None of theseAns : (A)

33. According to Urwick, where the work is of a more simple and routine nature, the span of control varies from—(A) 9 to 12(B) 8 to 12(C) 7 to 9(D) 10 to 12Ans : (B)

34. Delegation of authority by a Sales Manger to his saleman is an example of—(A) Upward delegation(B) Sideward delegation(C) Downward delegation(D) None of theseAns : (C)

35. A virtual organisation is—(A) One which has profit as the major goal(B) One in which leadership always tends to fulfil psychological needs of the subordinates(C) A small, core organization that outsources major busi-ness functions(D) One which has concern of the employees as its top priorityAns : (C)

36. The most effective means of citizen's control over administration is—(A) Election(B) Pressure Groups(C) Advisory Committees(D) Public OpinionAns : (A)

37. The Santhanam Committee on prevention of corruption was appointed in—(A) 1961(B) 1964(C) 1963(D) 1962Ans : (D)

38. The first country in the world to introduce the right to information was—(A) Norway

Page 54: Que

(B) USA(C) Sweden(D) FinlandAns : (C)

39. What is meant by the Doctrine of State Immunity ?(A) The State is immune to being sued(B) The State can be sued but not in the national courts(C) The State cannot be sued in its own court without its consent(D) None of the aboveAns : (A)

40. 'Habeas Corpus' literally means—(A) To have the body of(B) To command(C) To prohibit(D) None of the aboveAns : (A)

41. The system of 'Rule of Law' was propounded by—(A) A. V. Dicey(B) Lowell(C) W. F. Willoughby(D) H. FinerAns : (A)

42. Who says that "Power corrupts and absolute power corrupts absolutely" ?(A) L. D. White(B) Lord Beveridge(C) Lord Acton(D) Josiah StampAns : (C)

43. The Public Accounts Committee of Parliament in India consists of—(A) 15 members(B) 22 members(C) 25 members(D) 30 membersAns : (B)

44. If a public servant imposes upon the citizens duties and obligations which are not required by law, itcan lead to—(A) Error of law(B) Abuse of power(C) Error of authority(D) Error of fact findingAns : (B)

45. Of which of the following administrative systems is the absence of judicial review a feature ?1. USA2. UK3. France4. India(A) 3(B) 2 and 3(C) 1 and 3(D) 2Ans : (B)

46. The present constitution of the USA was adopted in—(A) 1786(B) 1778(C) 1787

Page 55: Que

(D) 1789Ans : (C)

47. 'Grand Corps' in French Civil Service stands for—(A) Senior level of specialist administrators(B) Senior level of generalist administrators(C) Senior level of generalistcum-specialist administra-tors(D) Senior level of defence administratorsAns : (C)

48. In France, the term of the President is—(A) Four years(B) Five years(C) Six years(D) Seven yearsAns : (D)

49. In England, the Cabinet must resign from office as soon as it loses the confidence of—(A) The King or the Queen(B) The House of Commons(C) The House of Lords(D) All of theseAns : (B)

50. Under their service rules, the British Civil servants—(A) Are required to be neutral in politics(B) Can be partisan(C) Can be partly neutral and partly partian(D) Can pursue active party politicsAns : (A)

1. The term 'Performance Budget' was coined by—(A) Administrative Reforms Commission of India(B) Second Hoover Commission of USA(C) Estimates Committee of India(D) First Hoover Commission of USAAns : (D)

52. During passing of budget in the Parliament 'Guilotine' is applied to those demands which are—(A) Discussed and approved(B) Discussed but not approved(C) Discussed and reduced(D) Not discussed for want of timeAns : (D)

53. Audit of State Government is—(A) A state subject(B) A union subject(C) In the concurrent list(D) None of theseAns : (B)

54. In which year was the Committee on Public Undertakings constituted by the Lok Sabha ?(A) 1953(B) 1956(C) 1963(D) 1964Ans : (D)

55. The number of demands in the general budget for civil expenditure is—(A) 103(B) 106(C) 102(D) 109

Page 56: Que

Ans : (A)

56. The rule of lapse means—(A) All appropriations voted by the legislature expire at the end of the financial year(B) All pending bills in Parliament lapse with its prorogation(C) The demand for grants of a ministry lapse with criticism of its policy by the opposition(D) The appropriation bill lapses if it is not returned by the Rajya Sabha within 14 daysAns : (A)

57. The C & AG of India does not audit the receipts and expenditure of—(A) Municipal undertakings(B) State Governments(C) Government companies(D) Central GovernmentAns : (A)

58. The role of the Finance Commission in Central-State fiscal relations has been undermined by—(A) The State Governments(B) The Zonal Councils(C) The Planning Commission(D) The Election CommissionAns : (A)

59. The Railway Budget was separated from the Central Budget in the year—(A) 1920(B) 1921(C) 1922(D) 1923Ans : (B)

60. The Chairman of the Public Accounts Committee of the Parliament is appointed by—(A) Speaker of Lok Sabha(B) Prime Minister of India(C) President of India(D) Chairman of Rajya SabhaAns : (A)

61. The question asked orally after the question hour in the House is called—(A) Supplementary question(B) Short notice question(C) Starred question(D) Unstarred questionAns : (C)

62. The economy cut motion seeks to reduce the proposed expenditure of a demand for grant by—(A) Rupees one hundred(B) Rupee one(C) A specified amount(D) An unspecified amountAns : (C)

63. Who was the Chairman of the Economic Reforms Commission (1981-84) ?(A) Dharam Vira(B) L. K. Jha(C) K. Hanumanthiya(D) B. G. DeshmukhAns : (B)

64. In which year was the decision of separating accounting function from audit function taken by theCentral Government for its financial administration ?(A) 1950(B) 1965(C) 1976(D) 2000Ans : (C)

Page 57: Que

65. In which recruitment, in a system for higher position is open to all the qualified candidates who maywish to apply is known as ?(A) Direct recruitment(B) Recruitment by promotion(C) Ordinary recruitment(D) Passive recruitmentAns : (A)

66. 'Position classification' is the classification of—(A) Salaries(B) Duties(C) Departments(D) Personal status of incumbentsAns : (B)

67. The civil service was defined as "professional body of officials, permanent, paid and skilled" by—(A) Herman Finer(B) O. G. Stahl(C) Felix Nigro(D) E. N. GladdenAns : (A)

68. A new All India Service can be created by—(A) An amendment of the constitution(B) An executive order(C) A resolution under Article 312 of the Constitution(D) A statuteAns : (C)

69. The British concept of Civil Service neutrality is laid down by—(A) Fulton Committee(B) Assheton Committee(C) Masterman Committee(D) Northcote-Trevelyn CommitteeAns : (C)

70. The 'spoils system' in the USA began during the period of—(A) Jefferson(B) Jackson(C) Washington(D) AdamsAns : (B)

71. Promotion in Civil Services indicates—(A) Such changes in the situation which indicate difficult work and more important responsibility(B) Change in the place of work(C) Transfer of work from field to Headquarters(D) Always an increase in payAns : (A)

72. Which of the following is not an All India Service ?(A) Indian Foreign Service(B) Indian Administrative Service(C) Indian Forest Service(D) Indian Police ServiceAns : (A)

73. The Union Public Service Commission of India has been established under the—(A) Article 315(B) Article 320(C) Article 325(D) Article 335Ans : (A)

Page 58: Que

74. Which of the following reports deals with the relations between the specialists and generalists ?(A) Haldane Committee Report(B) Sarkaria Commission Report(C) Fulton Committee Report(D) Kothari Committee ReportAns : (C)

75. Reservation for the scheduled castes and scheduled tribes in the services has been provided in theIndian Constitution under—(A) Article 315(B) Article 335(C) Article 365(D) Article 375Ans : (B)